You are on page 1of 99

Neurology revision-2015 april

1-A 65-year-old patient has had Parkinson’s disease for 10 years and is
currently taking benztropine 2 mg every day, pramipexole 1.5 mg 3 times
a day, and carbidopa/levodopa 25/100 mg 4 times a day. His wife claims
that he is complaining of seeing spiders and bugs running across the floor.
The first thing to do is:
A) Add clozapine
B) Add quetiapine
C) Taper and stop the benztropine
D) Taper and stop the carbidopa/levodopa
Ans:c
_______________________________________________________
2-what antiepileptic is associated with vision loss
Vigabatrin
______________________________________________________
3-T or F
High-protein diets decrease absorption of carbidopa-levodopa
Ans:true
______________________________________________________
4. B.V. is a 28-year-old woman brought to your emergency department
for treatment of status epilepticus. She receives lorazepam 4 mg
intravenously with subsequent seizure cessation. Which medication is the
best next treatment step for B.V.?
A. Topiramate.

1 By :Amera Elfeky Team april 2015


Neurology revision-2015 april

B. Phenytoin.
C. Zonisamide.
D. Diazepam.
4. Answer: B
In general, medications to treat status epilepticus should be in parenteral
formulation to facilitate rapid administration. Once the seizures of status
epilepticus have been stopped, a second, long-acting drug should be
started to prevent seizure recurrence. Medications typically used for this
purpose include phenytoin, fosphenytoin, phenobarbital, and
(sometimes) valproic acid. There is no need to administer another
benzodiazepine because the seizure activity has ceased.
_____________________________________________________
5-Which is true regarding fatigue in MS?
A) Fatigue occurs rarely in MS.
This is the correct answer.
B) The treatment of fatigue often is overlooked in MS patients.
C) Amantadine is useful orally and intrathecally to treat fatigue.
D) It is often helpful to treat for sleep improvement.
Ans:b
______________________________________________
6-A 57-year-old man with a history of end-stage renal disease and
epilepsy is currently taking phenytoin 300 mg daily for seizure control. His
most recent total phenytoin level was 12.4 mg/L (49.1 µmoI/L), and his
albumin level is 2.7 g/dL (27 g/L).
On the basis of this patient's current serum phenytoin concentration,
which adjustment should be made to his regimen?
A- Increase the phenytoin dose to ensure control of his seizures.

2 By :Amera Elfeky Team april 2015


Neurology revision-2015 april

B- Maintain the total daily dose, but recommend taking the medication as
100 mg three times daiy to ensure absorption.
C- Decrease the dose to reduce the risk of adverse effects.
D- Make no change to his regimen at this time
Ans: c- due to hypoalbuminemia so exx free phenytoin
_________________________________________________________
7-Which of the following is used in the treatment of an acute
attack/relapse of MS?
A) Oral immunoglobulin
This is the correct answer.
B) High-dose intravenous methylprednisolone
C) High-dose oral prednisone
D) Mitoxantrone
Ans:b
_______________________________________________________
8-A patient at annual checkup has been well controlled with phenytoin
oral capsules (300mg/d) for 7 months – i.e. no seizures to report. A
routine level is drawn and reported at 7.6 mcg/ml. After correcting for a
low albumin level you find it is actually 9.8 mcg/ml. The BEST
recommendation would be:
a. increase the dose in response to the corrected phenytoin level
b. switch the patient to oral phenytoin acid tablets at 300 mg/d as this
represents a higher dose of phenytoin base
c. decrease the dose as the corrected level is higher than the reported
value
d. have the level re-drawn because the patient is not yet at steady state
e. make no changes to the current regimen
3 By :Amera Elfeky Team april 2015
Neurology revision-2015 april

ans:e
(A is incorrect because although the patient's phenytoin level is not wihin
the accepted therapeutic range (10-20), there is no need to make
adjustments at this time as the patient is well controlled. B again
represents an increase in dose, and this recommendation wouldn't make
practical sense. C is incorrect as the corrected phenytoin level is a truer
level in relation to a low albumin (phenytoin is highly protein bound and a
free level may be indicated in this case). D is incorrect as the patient is
certainly at steady state by this time.)
__________________________________________________________
9. M.K. is a 44-year-old woman with right-sided headaches of moderate
intensity that are accompanied by severe nausea and vomiting. Which
triptan is best to treat M.K.’s migraine headaches?
A. Almotriptan.
B. Naratriptan.
C. Rizatriptan.
D. Sumatriptan.
. Answer: D
Sumatriptan is available as an injectable and as a nasal spray and would
be more appropriate to use in a patient with severe nausea and vomiting.
Zolmitriptan is available as a nasal spray. The other triptans are available
only in oral preparation
________________________________________________________
10-A 45-year-old woman has incapacitating migraines that occur about
twice weekly. Although she gets some relief from ergot alkaloid therapy,
she states she has to cancel plans at least once every few weeks. Her
medical history is significant for hypertension, and her medications
include lisinopril and ergotamine as needed.
Which is the most appropriate recommendation for this patient?

4 By :Amera Elfeky Team april 2015


Neurology revision-2015 april

A Daily use of sublingual ergotamine


B Frovatriptan orally daily before and during menses
C Acebutolol orally daily
D Topiramate orally daily
Answer D is correct. Topiramate is considered a medication with
established efficacy in migraine prevention because many clinical trials
versus placebo have shown a lower monthly frequency of migraines.
Other studies have shown efficacy similar to valproate or propranolol.
Ergotamine is not preferred for chronic use, and it imay be a problem in
patients with uncontrolled hypertension (Answer A). Frovatriptan has
established efficacy in preventing menstrual-related migraines; however,
the patient's history does not suggest her migraines are related to her
menstrual cycle (Answer B). β-Blockers with intrinsic sympathomimetic
activity (e.g., acebutolol) are ineffective for migraine prevention (Answer
C).
_________________________________________________
11. RD is a 19-year-old man who is shopping for greeting cards in the
pharmacy where you work. As you watch, he falls to the floor and his
body stiffens, then his muscles begin to contract rhythmically. You ask
one of the technicians to call 911, and soon an ambulance arrives. When
he arrives at the ED, the first medication they should give to stop his
seizure is which one of the following?
(A) diazepam
(B) lorazepam
(C) phenobarbital
(D) pentobarbital
11. Answer: B.
Lorazepam is the drug of choice for treatment of status epilepticus.
Phenobarbital, phenytoin, or fosphenytoin should be administered
immediately after the lorazepam to prevent further seizures.
5 By :Amera Elfeky Team april 2015
Neurology revision-2015 april

________________________________________________________
12. R.P. is a 25 Yo actress with simple partial seizures. She takes
topiramate. Which of the following adverse effects may be problematic
for her?
a. amenorrhea
b. metabolic alkalosis
c. word-finding difficulties
d. livedo reticularis
. Answer c. word finding difficulties. Topiramate can cuase acidosis not
alkalosis. It has some carbonic anhydrase inhibitor activity
______________________________________________________
13. L.L. is a 47-year-old man with Parkinson disease. He takes
carbidopa/levodopa 50/200 orally four times/ day. He recently noticed an
involuntary twitching movement of his left foot. Which is the best therapy
for L.L.’s dyskinesia?
A. Add ropinirole.
B. Add selegiline.
C. Increase carbidopa/levodopa.
D. Decrease carbidopa/levodopa
61. Answer: D
Treatment of dyskinesias includes decreasing the levodopa dose,
removing selegiline or dopamine agonists from the drug regimen, or
adding amantadine.
_____________________________________________________________
14. L.S. is a 72-year-old woman with a medical history of hypertension,
type 2 diabetes mellitus, renal failure, and atrial fibrillation. She presents
to the anticoagulation clinic for her initial visit. Which best reflects her
target INR?
6 By :Amera Elfeky Team april 2015
Neurology revision-2015 april

A. 1.5.
B. 2.0.
C. 2.5.
D. 3.0
. Answer: C
The target INR for a patient younger than 75 years with hypertension and
diabetes mellitus is 2.5.
___________________________________________________________
15. A 46 year old male with a history of epilepsy has been transported to
the ER in status epilepticus. Lorazepam has been administered but the
patient is still seizing. The patient's wife states that her husband has not
been feeling well and has not been taking his anti-seizure medications,
nor has he been eating or drinking much. The ER nurse is having difficulty
placing an IV line which he thinks may be due in part to volume depletion.
The ER physician calls you to recommend an anti-seizure medication that
can be given intramuscularly. Which one of the following would be most
appropriate?
A. levetiracetam
B. phenytoin
C. phenobarbital
D. valproic acid
. answer C.
Phenobarbital is formulated for IM use. IM administration of
levetiracetam has not been tested. Phenytoin has erratic absorption when
administered IM and injections are painful due to its high pH. Valproic
acid can cause tissue necrosis when given IM.
_______________________________________________________

7 By :Amera Elfeky Team april 2015


Neurology revision-2015 april

16. S.R. is a 54-year-old female homemaker with squeezing, band like


headaches that occur three or four times/ week. She rates the pain of
these headaches as 7 of 10 and finds acetaminophen, aspirin, ibuprofen,
naproxen ketoprofen, and piroxicam only partly effective. She wishes to
take a prophylactic drug to prevent these tension headaches. Which is
best for prophylaxis of her headaches?
A. Propranolol.
B. Valproic acid.
C. Amitriptyline.
D. Lithium.
. Answer: C
Amitriptyline is effective as prophylaxis for tension headaches. β-Blockers
and valproic acid are usually used for migraine headache prophylaxis, and
lithium is used for prophylaxis of cluster headaches.
________________________________________________________
17. Which patient situation would be an appropriate reason to consider
discontinuation of an anticonvulsant medication?
A. An abnormal electroencephalogram (EEG) during treatment with an
antiepileptic drug
B. Multiple seizure types consistent with an epilepsy syndrome
C. Patient having been seizure free for 2–5 years
D. Patient having taken an anticonvulsant medication for 5 years
. C. Patient having been seizure free for 2–5 years
Antiepileptic therapy may not need to be continued indefinitely.
Antiepileptic drug withdrawal may be an option for patients who have
been seizure free for 2–5 years and have a single type of seizure, a normal
neurologic examination and IQ, and an electroencephalogram (EEG) that
is normal with treatment.

8 By :Amera Elfeky Team april 2015


Neurology revision-2015 april

___________________________________________________________
18. D.S. is a 49-year-old male computer programmer who describes
lancinating right-eye pain and tearing several times a day for 2–3 days in a
row. He will have no episodes for 2–3 weeks but then will have recurrent
episodes. In the office, he receives oxygen by nasal cannula during an
episode, and his pain is relieved. He has a diagnosis of cluster headaches.
Which drug is best for prophylaxis of his headaches?
A. Propranolol.
B. Valproic acid.
C. Amitriptyline.
D. Lithium.
. Answer: D
Lithium is a prophylactic agent for cluster headaches. β-Blockers and
valproic acid are usually used for migraine headache prophylaxis.
Amitriptyline is useful for migraine and tension headaches.
__________________________________________________________
19.S.F. is a 33-year-old African American woman of Cuban descent living
in the Miami area. This morning, her right leg became progressively
weaker over about 3 hours. She was previously healthy except for a
broken radius when she was 13 years old and a case of optic neuritis
when she was 25 years old. Which is the best method for treating S.F.’s
exacerbation?
A. Interferon beta-1a.
B. Glatiramer acetate.
C. Mitoxantrone.
D. Methylprednisolone.
Answer :
===

9 By :Amera Elfeky Team april 2015


Neurology revision-2015 april

20. Which therapy is best for S.F. to prevent further exacerbations?


A. Interferon beta-1a.
B. Interferon beta-1b.
C. Glatiramer acetate.
D. Any of the above.
Answer D
======
21. S.F. elects to start interferon beta-1b and wants to know whether she
can prevent or minimize some of the adverse effects. Which is the best
advice?
A. Always give the injection at the same time of day.
B. Lie down for 2 hours after the injection.
C. Rotate injection sites.
D. Use a heating pad on the injection sites.
21. Answer: C
Rotating the injection sites for the self-injections is a good strategy for
preventing injection site reactions. Other strategies that might help
prevent these reactions are icing the injection site before injection and
bringing the drug to room temperature. The injections should be
administered at about the same time of day, but this is not a strategy for
preventing adverse effects.
_________________________________________________________
22. Which of the following anti-epileptic drugs is NOT a controlled
substance?
A. lacosamide
B. pregabalin
C. topiramate
10 By :Amera Elfeky Team april 2015
Neurology revision-2015 april

Ans:c
___________________________________________________________
23-R.B. is a 37-year-old woman with a long-standing history of focal
seizures. She currently takes
carbamazepine 400 mg orally 3 times/day with good seizure control.
Recently, she received a
diagnosis of multiple sclerosis. Which one of the following medications is
R.B. best advised to
avoid?*
A. Dalfampridine.
B. Mitoxantrone.
C. Glatiramer acetate.
D. Beta interferon-1a
Ans:a
what to avoid
so it will be dalfamepridine as it cause seizures as a side effect and the
patient already is documented to have focal seizure
__________________________________________________________
24. PZ is a 84-year-old, African-American man with the following health
conditions: hypertension, hypercholesterolemia, Raynaud’s disease, and
aspirin allergy (wheezing). His current medications include: amlodipine 5
mg PO QD and fluvastatin 40 mg PO QHS. He experienced sudden onset of
right-sided paralysis about 1 hour ago. He waited 15 minutes after the
paralysis began, but experienced no improvement. He called his neighbor
who immediately called 911 and arranged for transport to the emergency
department (ED). In the ED, he is whisked off to the CT scanner where
they find no evidence of intracerebral hemorrhage. His vital signs are as
follows: BP 148/92 mm Hg, heart rate 86 BPM, R 21/minute. Is PZ a

11 By :Amera Elfeky Team april 2015


Neurology revision-2015 april

candidate for tissue plasminogen activator treatment?


(A) yes
(B) no, too much time has elapsed since symptom onset
(C) no, his blood pressure is too high
(D) no, he has Raynaud’s disease
. Answer: A.
This patient is a good candidate for tissue plasminogen activator
treatment. His stroke symptoms began less than 3 hours ago. He has no
active bleeding or major bleeding risk. His blood pressure is >185/110 mm
Hg.
_______________________________________________
25-Concurrent administration of diazepam and phenobarbital may result
in _____________, so additional ICU monitoring may be needed.
A) Arrhythmia
B) Hypotension
C) Respiratory depression
D) Extravasation
Ans:c
___________________________________________________________
26. Which of the following anti-seizure medications may be taken with or
without food?
A. Zarontin
B. Neurontin
C. Gabitril
D. Tegretol XR
Ans: b...........Neurontin (gabapentin) may be taken with or without food.
Zarontin (ethosuximide) should be taken with food to minimize GI upset.
Rapid oral absorption of Gabitril (tiagabine) is associated with increased

12 By :Amera Elfeky Team april 2015


Neurology revision-2015 april

CNS side effects so administration with food is recommended to slow the


absorption rate. Some formulations of carbamazepine (such as Tegretol
XR) should be taken with food, while others may be taken with or without
food.
___________________________________________________________
27. Which one of the following anti-seizure medications is NOT renally
dosed?
A. valproic acid
B. gabapentin
C. oxcarbazepine
D. levetiracetam
. answer A.
Gabapentin is eliminated almost exclusively by the kidneys and
reductions in both dosing and frequency of administration are required in
patients with moderate-severe renal function (CrCL < 60). The active
metabolite of oxcarbazepine (MHD) undergoes renal elimination, though
dose adjustments are recommended only in the case of severe renal
dysfunction (CrCL < 30). Dose reductions are recommended in patients
taking levetiracetam who have a CrCL < 80. Though clearance of valproic
acid decreases by about 25% in patients with CrCL < 10 these patients are
typically on hemodialysis, which removes about 20% of valproic acid
concentrations; thus no dose adjustment is necessary.
________________________________________________
28.RM is a 68-year-old man who presents to the neurology clinic because
of concerns with his gait and decreased speech volume. He frequently
finds himself tripping when he walks and his wife is concerned that he will
fall and break his hip. On examination, he is found to have slowness of
movement when walking and difficulty maintaining his balance when his
shoulders are tugged from behind. He is difficult to hear, with the
examiner often asking him to repeat answers to questions. Otherwise his

13 By :Amera Elfeky Team april 2015


Neurology revision-2015 april

neurological and physical examinations are normal. RM is diagnosed with


Parkinson’s disease. Which one of the following medications would be the
best treatment choice for RM?
(A) ropinirole
(B) entacapone
(C) trihexyphenidyl
(D) primidone
======
29. Despite your recommendation, the physician chooses to place RM
(from question No. 2) on carbidopa/levodopa 25 mg/100 mg PO three
times daily (TID). RM does well on this regimen for a year. When you see
him back in clinic at that time, he has noticed a new problem. He thinks
his carbidopa/levodopa is not lasting as long as it did previously. He
notices that his Parkinson's disease symptoms return about 1 hour before
it is time to take another dose. Which one of the following should be your
assessment and recommendation of this problem?

(A) this is wearing off; it can be improved by giving a larger dose

(B) this is wearing off; it can be improved by giving the doses closer
together

(C) this is on-off; it can be improved by giving a larger dose

(D) this is on-off; it can be improved by giving the doses closer together
28. Answer: A.
Ropinirole is a dopamine agonist used in the treatment of Parkinson’s
disease. Entacapone is a catechol-O-methyl transferase inhibitor used in
the treatment of Parkinson’s disease, but it must be used in combination
with carbidopa/levodopa. Trihexyphenidyl is an anticholinergic used in
14 By :Amera Elfeky Team april 2015
Neurology revision-2015 april

the treatment of Parkinson’s disease, but it only controls the symptom of


tremor. Primidone is used for the treatment of essential tremor, not
Parkinson’s disease.
29. Answer: B.
Wearing off is a return of Parkinson's disease symptoms prior to the end
of the carbidopa/levodopa dosing interval. It is corrected in one of three
ways:
1) giving the doses closer together,
2) changing to a controlled release formulation of carbidopa/levodopa, or
3) adding entacapone. On-off is a return of Parkinson's disease symptoms
at any time during the dosing interval.
_______________________________________________

30. M.R., a 34-year-old pharmacist, has throbbing right-sided headaches.


She experiences nausea, photophobia, and photophobia with these
headaches, but no aura. She usually has headaches two times/month. She
is hypertensive and morbidly obese. She takes an ethinyl
estradiol/progestin combination oral contraceptive daily and
hydrochlorothiazide 25 mg/day orally. She has a diagnosis of migraine
headaches. Which medication is best for prophylaxis of her headaches?
A. Propranolol.
B. Valproic acid.
C. Amitriptyline.
D. Lithium.
Ans:a-A β-blocker is a good choice for a patient with the coexisting
condition of hypertension. Valproic acid and amitriptyline could both
increase weight gain in a morbidly obese patient. Lithium is used for
prophylaxis of cluster headaches.
_________________________________________________________

15 By :Amera Elfeky Team april 2015


Neurology revision-2015 april

31. A patient at annual checkup has been well controlled with phenytoin
oral capsules (300mg/d) for 7 months – i.e. no seizures to report. A
routine level is drawn and reported at 7.6 mcg/ml. After correcting for a
low albumin level you find it is actually 9.8 mcg/ml. The BEST
recommendation would be:

a. increase the dose in response to the corrected phenytoin level

b. switch the patient to oral phenytoin acid tablets at 300 mg/d as this
represents a higher dose of phenytoin base

c. decrease the dose as the corrected level is higher than the reported
value

d. have the level re-drawn because the patient is not yet at steady state

e. make no changes to the current regimen


. answer e.
make no changes to the current regimen (A is incorrect because although
the patient's phenytoin level is not wihin the accepted therapeutic range
(10-20), there is no need to make adjustments at this time as the patient
is well controlled. B again represents an increase in dose, and this
recommendation wouldn't make practical sense. C is incorrect as the
corrected phenytoin level is a truer level in relation to a low albumin
(phenytoin is highly protein bound and a free level may be indicated in
this case). D is incorrect as the patient is certainly at steady state by this
time.)
______________________________________________________

16 By :Amera Elfeky Team april 2015


Neurology revision-2015 april

32. C.A., a 57-year-old white man who just retired from the New York City
Fire Department, has been experiencing tremors in his right hand that
have become progressively worse for the past 6 months. He has difficulty
walking. He also has backaches and no longer plays golf. In addition, he is
losing his sense of taste. He is given a diagnosis of Parkinson disease.
Which is the best treatment for this man?
A. Trihexyphenidyl.
B. Entacapone.
C. Apomorphine.
D. Ropinirole.
. Answer: D
Ropinirole, a direct dopamine agonist, is a good choice for initial
treatment in a patient with Parkinson disease. Trihexyphenidyl would
control his tremor but would not help his difficulty walking, which likely
represents bradykinesia. Entacapone is a catechol-O-methyl-transferase
inhibitor; it should only be used in conjunction with carbidopa/levodopa.
Apomorphine is for severe on-off symptoms.
______________________________________________________
33. L.R. is a 78-year-old man who presents to the emergency department
for symptoms of right-sided paralysis. He states that these symptoms
began about 5 hours ago and have not improved since then. He also has
hypertension, benign prostatic hypertrophy, diabetes mellitus, erectile
dysfunction, and osteoarthritis. Which is the most accurate assessment of
L.R.’s risk factors for stroke?
A. Erectile dysfunction, age, osteoarthritis.
B. Sex, diabetes mellitus, osteoarthritis.
C. Benign prostatic hypertrophy, diabetes mellitus, age, sex.
D. Age, diabetes mellitus, sex, hypertension.
=====

17 By :Amera Elfeky Team april 2015


Neurology revision-2015 april

34. Is L.R. a candidate for tissue plasminogen activator for treatment of


stroke? Which option is the best response?
A. Yes.
B. No, he is too old.
C. No, his stroke symptoms began too long ago.
D. No, his diabetes mellitus is a contraindication for tissue plasminogen
activator.
=====
35. He was previously taking no drugs at home. Which choice is the best
secondary stroke prevention therapy for this patient?
A. Sildenafil.
B. Celecoxib.
C. Aspirin.
D. Warfarin.
33. Answer: D
Non-modifiable risk factors for stroke include age, race, and male sex.
Somewhat modifiable risk factors include hypercholesterolemia and
diabetes mellitus. Modifiable stroke risk factors include hypertension,
smoking, and atrial fibrillation. Less well-documented risk factors include
obesity, physical inactivity, alcohol abuse, hyperhomocysteinemia,
hypercoagulability, hormone replacement therapy, and oral
contraceptives. Modification of risk factors, if possible, may translate into
reduced stroke risk, which should be a focus of all stroke prevention
plans.

34. Answer: C
Contraindications to administering tissue plasminogen activator for stroke
include intracranial or subarachnoid bleeding (or history), other active

18 By :Amera Elfeky Team april 2015


Neurology revision-2015 april

internal bleeding, recent intercranial surgery, head trauma, BP greater


than 185/110 mm Hg, seizure at stroke onset, intracranial neoplasm,
atrioventricular malformation, aneurysm, active treatment with warfarin
or heparin, and platelet count less than 100,000. There is no upper limit
on age. Until recently, there was a strict 3-hour limit for treating strokes.
A recent study indicates this limit may be increased safely to administer
tissue plasminogen activator 4.5 hours after symptom onset.

35. Answer: C
All patients experiencing a stroke should be placed on a drug to prevent
future events. Appropriate choices include the following: aspirin,
ticlopidine, cilostazol, clopidogrel, dipyridamole-aspirin, and warfarin. If
the patient has atrial fibrillation, he/she should be treated with warfarin,
dabigatran, or rivaroxaban. If the patient does not have atrial fibrillation,
warfarin offers no benefit but has considerable risk compared with
aspirin. Otherwise, any of these drugs are reasonable choices.
____________________________________________________________
36. C.P. is a 69-year-old man given a diagnosis of Parkinson disease 7
years ago. He states that he is most bothered by his bradykinesia
symptoms. On examination, he also has a pronounced tremor, postural
instability, and masked facial expression. He currently takes
carbidopa/levodopa/entacapone 25 mg/100 mg/200 mg orally four
times/day, ropinirole 1 mg orally three times/day, and selegiline 5 mg
orally two times/day. He has no drug allergies. He also describes a
worsening of his Parkinson disease symptoms, which fluctuate randomly
during the day. He has developed a charting system for his symptoms
during the day, and no relationship seems to exist with the time he is
scheduled to take his doses of carbidopa/levodopa. Which condition best
describes C.P.’s fluctuating Parkinson disease symptoms?
A. Wearing-off.
B. On-off.

19 By :Amera Elfeky Team april 2015


Neurology revision-2015 april

C. Dyskinesia.
D. Dystonia.
=========
37. For his symptoms, C.P. (from question 14) is given a prescription for
apomorphine. Which is the most accurate statement regarding this drug?
A. He must be trained on self-injection techniques with saline, but he can
administer his first dose of apomorphine at home when he needs it.
B. He should not take apomorphine if he is allergic to penicillin.
C. If he does not take a dose for more than 1 week, he should begin with a
loading dose with his next injection.
D. It may cause severe nausea and vomiting
36. Answer: B
Wearing-off is the return of symptoms before the next dose. It has a
definite pattern, whereas on-off is unpredictable. Dyskinesias and
dystonias are long-term adverse effects of carbidopa/levodopa

37. Answer: D
The first dose of apomorphine must be given in a clinic setting. The
patient should not take apomorphine if he is allergic to sulfa. The dose
should undergo retitration if he has not taken apomorphine for 1 week.
Apomorphine causes severe nausea and vomiting
_____________________________________________________
38. You are the pharmacist at a community pharmacy and receive a call
from M.W., a 64-year-old man recently given a diagnosis of atrial
fibrillation. He is concerned about his risk of having a stroke because his
friend, who also has atrial fibrillation, asked him what dose of warfarin he
is taking. M.W. called you because he is not taking warfarin and wants to
know whether he should. He has no other medical conditions and takes

20 By :Amera Elfeky Team april 2015


Neurology revision-2015 april

atenolol 50 mg/day orally for ventricular rate control. After encouraging


him to discuss this with his physician, which choice best describes what
you should tell him?
A. You need warfarin treatment to prevent a stroke.
B. You do not need warfarin, but you should take aspirin and clopidogrel.
C. You do not need drug therapy at this time.
D. Because you have atrial fibrillation, nothing can reduce your risk of
stroke.
. Answer: C
No therapy is an appropriate choice for this patient (CHADS2 score = 0)
because he is younger than 75 years and has no other risk factors, such as
hypertension or a prosthetic valve
______________________________________________________
39. An 18-year-old woman with a history of partial seizures is well
controlled on valproic acid. She admits to being sexually active with her
boyfriend and states that they have unprotected sex and that she wishes
to become pregnant. What is the best recommendation regarding her
seizure therapy?
A. Continue her current treatment regimen.
B. Switch her therapy to levetiracetam.
C. Switch her therapy to phenytoin.
D. Switch her therapy to gabapentin.
. answer B. Switch her therapy to levetiracetam.
Valproic acid and phenytoin are teratogenic and should be avoided, if
possible, in pregnancy. Gabapentin causes weight gain, which is not
desirable in this population. Levetiracetam is the safest choice
_________________________________________________________

21 By :Amera Elfeky Team april 2015


Neurology revision-2015 april

40. L.S. is taking carbidopa/levodopa 25 mg/100 mg orally four times/day


and trihexyphenidyl 2 mg orally three times/day for his Parkinson disease.
L.S.’s wife reports that he is often confused and experiences constipation;
he has trouble talking because of his dry mouth. Which change is best to
resolve these symptoms?
A. Increase carbidopa/levodopa.
B. Increase trihexyphenidyl.
C. Decrease carbidopa/levodopa.
D. Decrease trihexyphenidyl.
======
41. Six months later, L.S. returns to the clinic concerned that his
carbidopa/levodopa dose is wearing off before his next dose is due.
Which is best to suggest?
A. Increase the dose of carbidopa/levodopa.
B. Decrease the dose of carbidopa/levodopa.
C. Increase the dosing interval.
D. Decrease the dosing interval.
40. Answer: D
Anticholinergic drugs (benztropine and trihexyphenidyl) commonly cause
adverse effects such as confusion, dry mouth, urinary retention, and
constipation in older patients. Decreasing or eliminating these drugs may
resolve the difficulties.
41. Answer: D
Wearing-off phenomenon is the return of Parkinson disease symptoms
before the next dose. This problem can be resolved by giving doses more
often, administering the controlled-release formulation of carbidopa/
levodopa, or adding a catechol-O-methyl transferase inhibitor. The terms

22 By :Amera Elfeky Team april 2015


Neurology revision-2015 april

increase and decrease the dosing interval are often misinterpreted. To


increase the dosing interval means to give the doses further apart.
_______________________________________________________
42. R.H. is a 59-year-old man who presents to the emergency department
for new-onset left-sided weakness that began 6 hours ago. He has a
history of hypertension and coronary artery disease. His medication list
includes atenolol 50 mg/day orally, hydrochlorothiazide 25 mg/day orally,
and aspirin 325 mg/day orally. His vital signs include blood pressure (BP)
160/92 mm Hg, heart rate 92 beats/ minute, respiratory rate 14
breaths/minute, and temperature 38°C. The treatment team assesses this
patient for treatment with tissue plasminogen activator and asks for your
opinion. Which is the best reply given this information?
A. R.H. should be treated with tissue plasminogen activator.
B. R.H. should not be treated with tissue plasminogen activator because
the onset of his stroke symptoms was 6 hours ago.
C. R.H. should not be treated with tissue plasminogen activator because
he has hypertension.
D. R.H. should not be treated with tissue plasminogen activator because
he takes aspirin.
========
43. R.H. (from question 11) survives his stroke. As part of his discharge
treatment plan, you evaluate his risk factors for a second stroke. Which
medication for secondary stroke prevention is best to begin at this time?
A. Dipyridamole.
B. Enoxaparin.
C. Heparin.
D. Clopidogrel.
42. Answer: B

23 By :Amera Elfeky Team april 2015


Neurology revision-2015 april

Patients who can be treated within 3 hours of stroke symptom onset


should be considered for tissue plasminogen activator. A recent study
showed good outcomes without excess mortality in patients treated
within 4.5 hours of stroke onset; however, more exclusion criteria must
be applied. Uncontrolled hypertension (greater than 185/100 mm Hg) is a
contraindication to tissue plasminogen activator treatment. Active use of
heparin (with an elevated partial thromboplastin time) or warfarin (with
an elevated INR) is a contraindication, but use of aspirin is not. This
patient’s onset of stroke symptoms began 6 hours ago, so he is not
eligible for tissue plasminogen activator treatment.

43. Answer: D
All stroke survivors require secondary stroke prevention drugs. If a
patient claims to be adherent on aspirin when he sustained his first
stroke, a different drug is usually considered. Clopidogrel or
dipyridamole-aspirin would be an acceptable choice. Heparin and
enoxaparin are not suitable for long-term home use for secondary stroke
prevention.
______________________________________________________
44. P.J. is a 57-year-old man with an 8-year history of Parkinson disease.
His current drugs include carbidopa/ levodopa 50/200 orally four
times/day, entacapone 200 mg orally four times/day, and amantadine
100 mg three times/day. He presents to the clinic with a reddish blue
discoloration on his lower arms and legs. Which, if any, of his drugs is the
most likely cause of this condition?
A. Carbidopa/levodopa.
B. Entacapone.
C. Amantadine.
D. None; likely represents venous stasis.
44. Answer: C

24 By :Amera Elfeky Team april 2015


Neurology revision-2015 april

Amantadine can cause livedo reticularis, a condition in which the dilation


of capillary blood vessels and the stagnation of blood within these vessels
cause a mottled, reddish blue discoloration of the skin. This usually occurs
on the trunk and extremities; it is more pronounced in cold weather.
Although simple venous stasis could occur, livedo reticularis is more likely
in this patient.
________________________________________________________

45. A 60-year-old man presents to his neurologist for his second visit since
receiving a diagnosis of Parkinson disease. The patient is currently taking
carbidopa/levodopa 25/100 mg by mouth four times/day. On
examination, the neurologist records a blood pressure (BP) of 110/70 mm
Hg, a Mini-Mental State Examination (MMSE) of 25, and a geriatric
depression scale (GDS) of 5 and notes mild dyskinesia. The neurologist
decides to decrease carbidopa/levodopa to three times/day after a
discussion with the patient, and a follow-up appointment is scheduled in
4 weeks. Which is an American Academy of Neurology quality measure,
which has not been met during this patient visit?
A. Assessment for psychiatric disorders
B. Assessment for autonomic dysfunction
C. Assessment for cognitive impairment or dysfunction
D. Assessment for medication-related motor complications
45. B. Assessment for autonomic dysfunction
Answer B is correct. According to the American Academy of Neurology
quality measures, patients with Parkinson disease should be asked about
and assessed for autonomic dysfunction (e.g., orthostatic hypotension,
constipation, incontinence).
____________________________________________________
46. J.H. is a 42-year-old man with complex partial seizures for which he
was prescribed topiramate. He has been increasing the dose of
25 By :Amera Elfeky Team april 2015
Neurology revision-2015 april

topiramate every other day according to instructions from his primary


care provider. He comes to the pharmacy where you work but seems a
little confused and has difficulty finding the words to have a conversation
with you. Which is the best assessment of J.H.’s condition?
A. Discontinue topiramate; he is having an allergic reaction.
B. Increase his topiramate dose; he is having partial seizures.
C. Slow the rate of topiramate titration; he is having psychomotor
slowing.
D. Get a topiramate serum concentration; he is likely supratherapeutic.10.
Answer: C
Psychomotor slowing is a very troublesome adverse effect for many
patients initiated on topiramate. It usually manifests as difficulty
concentrating, difficulty thinking, word-finding difficulties, and a feeling
of slowness of movement. The usual dosage titration for topiramate calls
for increasing the dose every week. This patient has been increasing the
dose of topiramate every other day. Because psychomotor slowing is
related to the speed of titration, this makes slowing the rate of titration
the most probable answer. Partial seizures could present as confusion;
however, they are unlikely to be a continuous condition
______________________________________________
47-Dr. K. would like assistance choosing an antiepileptic
drug (AED) that will not interfere with his
transplant patient’s cyclosporine. Which AED is
most appropriate to use in this patient (assuming
all will provide adequate seizure control)?
A. Carbamazepine.
B. Lacosamide.
C. Oxcarbazepine.

26 By :Amera Elfeky Team april 2015


Neurology revision-2015 april

D. Phenytoin.
Answer: B
The most correct answer is lacosamide. Cyclosporine
is a substrate of the CYP3A4 isoenzyme. Carbamazepine
is not the best option because it is a potent inducer
of the CYP3A4 isoenzyme. Oxcarbazepine is a moderate
enzyme inducer/inhibitor at high doses of the drug.
Oxcarbazepine is metabolized by CYP3A4, so again,
the serum concentration would be lowered for cyclosporine.
Phenytoin is also not preferred for the same
reasons (i.e., it is a potent enzyme inducer of CYP3A4).
Carbamazepine, oxcarbazepine, and phenytoin can be
given; however, extensive monitoring of the cyclosporine
levels and signs of organ rejection would be recommended.

____________________________________________________________
48-A 27-year old female is admitted unresponsive and seizing with a
presumed diagnosis of viral encephalopathy. She was admitted to the ICU
and intubated. At an outlying hospital, she received diazepam 0.25 mg/kg
IV x 3. Upon arrival to your unit, she received phenytoin 20 mg/kg IV and
is still seizing.
Which of the following would be most correct?

A) Fosphenytoin 15 PE/kg

27 By :Amera Elfeky Team april 2015


Neurology revision-2015 april

B) Valproate 30 mg/kg

C) Midazolam 0.2 mg/kg

D) Phenobarbital 20 mg/kg

49-If the above treatment failed, which of the following would you not
recommend?

A) Fosphenytoin 15 PE/kg IV

B) Valproate 30 mg/kg IV

C) Pentobarbital 20 mg/kg IV, infusion 1 mg/kg/h

D) Phenobarbital 10 mg/kg IV
Ans:d-a
_____________________________________________________

50. W.S. is a 57-year-old man initiated on rasagiline for treatment of his


newly diagnosed Parkinson disease. He develops a cough, body aches,
and nasal congestion. Which medication is best to treat W.S.’s symptoms?

28 By :Amera Elfeky Team april 2015


Neurology revision-2015 april

A. Guaifenesin.
B. Dextromethorphan.
C. Tramadol.
D. Pseudoephedrine
. Answer: A
Because of the MAO inhibition induced by rasagiline, patients should not
take meperidine, propoxyphene, tramadol, methadone,
dextromethorphan, sympathomimetics, fluoxetine, or fluvoxamine.
Guaifenesin can be safely taken in this situation.

__________________________________________________
51-there's Q about the cheapest alternative to gabapentin
a)pregabalin
b)tramadol
c)topiramate
d)duloxetine
ans:b
__________________________________________________
52. Which of the following can increase the risk of serious rash with
lamotrigine?

A. concomitant use of valproic acid

B. age > 65

C. slow dose escalation

29 By :Amera Elfeky Team april 2015


Neurology revision-2015 april

D. administration of an amino penicillin (such as amoxicillin) in the past


90 days
Ans:a
Valproic acid may increase the serum concentration of lamotrigine. The
dose of lamotrigine should be decreased significantly in a patient already
taking valproic acid. Patients should be monitored for increased toxic
effects of lamotrigine if valproic acid is initiated or increased. The
incidence of lamotrigine-associated rash is higher in children than in
adults. Rapid dose escalation appears to increase the risk. There are no
known interactions between lamotrigine and penicillins.
_____________________________________________________________
_________
53-Which AED does not interact with oral combination contraceptives?
A) Tiagabine
B) Lamotrigine
C) Valproic acid
D) Carbamazepeine
E) Oxcarbazepine
Ans:a
http://www.epilepsytoronto.org/pub/pdf/medications/MED-
valproicacid.pdf

____________________________________________________
54. Match the following anti-epileptic drugs with their unique side effects:

A. Carbamazepine

30 By :Amera Elfeky Team april 2015


Neurology revision-2015 april

B. Valproic acid

C. Pregabalin

D. Phenytoin

1. pedal edema

2. gingival hyperplasia

3. hyponatremia

4. Hyperammonemia
53. A. 3 B. 4 C. 1 D. 2
_____________________________________________________
55. Match the following anti-epileptic medications with their unique
pharmacokinetic features:

A. Carbamazepine

B. Oxcarbazepine

C. Valproic acid

D. Gabapentin

1. beta-oxidation

31 By :Amera Elfeky Team april 2015


Neurology revision-2015 april

2. autoinduction

3. renal elimination

4. Prodrug
A. 2 B. 4 C. 1 D. 3
Carbamazepine unique by autoinduction
Oxacarbazine is pro drug of Carbamazepine
Valboric acid is metabolized mainly by liver (beta oxidation)
Cause hepatotoxicity
Gabapentine is not metabolized renaly excreted
____________________________________________________________
56. Which one of the following anti-epileptic drugs does NOT have a
‫اﻟﺳﺑت‬
clinically important metabolite?

A. Carbamazepine

B. Lamotrigine

C. Oxcarbazepine

D. Primidone
. answer B.
Carbamazepine is metabolized to a 10,11-epoxide that has antiepileptic
activity. Oxcarbazepine produces 10-hydroxy carbamazepine (MHD)
which also has antiseizure effects. Primidone is metabolized to
phenobarbital. Lamotrigine produces no known active metabolites.
_______________________________________________________

32 By :Amera Elfeky Team april 2015


Neurology revision-2015 april

57-Which of the following characteristics make(s) a stroke patient


ineligible for intravenous thrombolysis?

This is the correct answer.


A) Hemorrhage seen on CT of the head
B) Blood pressure >170/90
C) Time of onset >3 hours
D) a and c
E) a, b, and c
Ans:a
__________________________________________________-
58-. Which one of the following is NOT considered an advantage of
fosphenytoin over phenytoin?

A. IM administration

B. improved solubility

C. faster administration time

D. no need for cardiac monitoring


58. answer D.
Although it is approved for IM use, IM administration of phenytoin is not
recommended due to pain on injection (due to its high pH) and erratic
absorption. Phenytoin can only be mixed in NS, whereas fosphenytoin is
soluble in saline and dextrose solutions. The recommended maximum
infusion rate fosphenytoin is 150 mg/min, which is three times faster than

33 By :Amera Elfeky Team april 2015


Neurology revision-2015 april

the recommended maximum phenytoin infusion rate. Cardiac monitoring


is still recommended due to risk of bradycardia.
__________________________________________________________

59-What would be the appropriate dose of oxcarbazepine in a patient


currently taking carbazepine 400 mg. PO BID?

A. 400 mg BID
B. 600 mg BID
C. 800 mg BID
D. There is no known dosing conversion from carbamazepine to
oxcarbazepine
B. When switching from carbamazepine to oxcarbazepine, the dose of
carbamazepine should be multiplied by a factor of 1.5.
_____________________________________________________

60. True or false:


a patient who develops a rash on one anti-epileptic drug has an increased
risk of developing a rash with other anti-seizure medications.
60 answer . True. 58% of patients who develop a rash with phenytoin will
also get a rash with carbamazepine. 40% of patients who get a rash with
carbamazepine will also get one with phenytoin, while 20-30% will get
one with oxcarbazepine. 80% of patients who experience a rash with
phenobarbital will get a rash with carbamazepine or phenytoin. Patients
with a history of developing a rash with other AEDs are more likely to get
one with lamotrigine as well.
______________________________________________________

34 By :Amera Elfeky Team april 2015


Neurology revision-2015 april

61-Which one of the following anti-epileptic drugs does NOT have a


clinically important metabolite?

A. Carbamazepine
B. Lamotrigine
C. Oxcarbazepine
D. Primidone
Ans:b
___________________________________________________
62-Which one of the following is NOT considered an advantage of
fosphenytoin over phenytoin?

A. IM administration
B. improved solubility
C. faster administration time
D. no need for cardiac monitoring
Ans:d
___________________________________________
63-Match the following for treatment of MS.

1. Can experience flu-like symptoms that usually dissipate after 2-3


months
2. Risk of progressive multifocal leukoencephalopathy
3. Cardiotoxicity is lifetime dose limiting toxicity
4. Used for acute relapse of MS

35 By :Amera Elfeky Team april 2015


Neurology revision-2015 april

a. Prednisone
b. Beta Interferons
c. Natalizumab
d. Mitoxantrone
1b
2c
3d
4a
_____________________________________________________
64-Which dopamine agonist is contraindicated with ondansetron?

A. Mirapex (pramipexole)
B. Requip (ropinirole)
C. Apokyn (apomorphine)
D. Azilect (rasagiline)
Ans:c
_____________________________________________________
65-An 85-year-old Caucasian female was brought to the emergency room
after falling in the kitchen. The event was witnessed by her husband. She
arrived at the ER 60 minutes after the onset of symptoms. She has a
history of atrial fibrillation for 1 year, hypertension for 50 years, and mild
congestive heart failure for 1 year.
Meds on admission: hydrochlorothiazide 25 mg daily, digoxin 0.125 mg
daily, aspirin 81 mg daily.
Review of systems: 5’, 100 lb, BP = 180/60, P = 85, EKG = a.fib

36 By :Amera Elfeky Team april 2015


Neurology revision-2015 april

Neurological exam: R-sided plegia, global aphasia, decreased alertness;


NIHSS = 23
CT (head): no evidence of hemorrhage or edema; consistent with ischemia
Based on the data above, is the patient eligible for thrombolytic therapy
with t-PA?

A) Yes
B) No

And:yes
___________________________________________________________
66-J.B. is an 8-year-old male who experiences absence seizures. Which
antiepileptic drug (AED) should not be used to treat this patient?
A) Lamotrigine
B) Valproic acid
C) Ethosuximide
D) Topiramate
E) Carbamazepine
Ans:e
_________________________________________________________

67-Which of the following medications has an elevated risk of


hepatoxicity in children?

a. oxcarbazepine

37 By :Amera Elfeky Team april 2015


Neurology revision-2015 april

b. valproate
c. topiramate
d. ethosuximide
ands:b
___________________________________________________________
68-Which of the following is true regarding fosphenytoin and phenytoin?
A) Phenytoin causes pruritis, whereas fosphenytoin does not.
B) Fosphenytoin does not cause arrhythmias, whereas phenytoin does.
C) A post-loading dose of phenytoin concentration can be drawn at 1 hour
following IV administration, whereas a post-loading dose of fosphenytoin
concentration can be drawn at 2 hours after a dose.
D) Phenytoin (50 mg/min) reaches concentrations more slowly than
fosphenytoin (50 PE/min).
Ans:c
__________________________________________________________
69-J.M. is a 55-year-old male with a partial seizure disorder who is
undergoing a liver transplant due to end-stage cirrhosis. Which AED
should be avoided?
A) Felbamate
B) Gabapentin
C) Pregabalin
D) Levetiracetam
E) Topiramate
Ans:a
_________________________________________________________

38 By :Amera Elfeky Team april 2015


Neurology revision-2015 april

70-Which of the following is not a side effect seen with interferon therapy
(interferon ß1a [Avonex, Rebif] or interferon ß1b [Betaseron])?

A) Flu-like symptoms
B) Depression
C) Chest tightness and facial flushing
D) Skin injection-site reactions
Ans:c
______________________________________________________
71-S.J. is a 48-year-old female of Southeast Asian heritage who suffers
from complex partial seizures. Which AED(s) is best avoided in this patient
until further testing can be done?

A) Topiramate
B) Phenytoin
C) Vigabatrin
D) Carbamazepine
E) Both b and d
CBZ, OXC and PHT, should be avoided in the B*1502 carrier and caution
should also be exercised for LTG.so answer is E
__________________________________________
72-All of the following AEDs can be involved in a drug interaction utilizing
the glucuronic pathway, except:
A) Pregabalin
B) Valproic acid

39 By :Amera Elfeky Team april 2015


Neurology revision-2015 april

C) Carbamazepine
D) Phenytoin
E) Oxcarbazepine
Ans:a
______________________________________________________
73-Tachyphylaxis is most commonly associated with which of the
following?
A) Midazolam
B) Ketamine
C) Phenytoin
D) Phenobarbital
Ans:d
_____________________________________________________
74-Which one of the following drug or drug classes is not used in the
acute treatment of migraine headaches?

A) Ergot alkaloids
B) Antidepressants
C) NSAIDs
D) Serotonin agonists
E) Acetaminophen
Ans:b
_______________________________________________________

40 By :Amera Elfeky Team april 2015


Neurology revision-2015 april

75-Which of the following is the most common adverse effect of the


ergotamine derivatives?

A) Painful extremitie
B) Peripheral ischemia
C) Nausea and vomiting
D) Continuous paresthesia
Ans:c
____________________________________________________
76-Which of the following is true regarding diazepam and lorazepam?
A) Lorazepam has a more rapid onset than diazepam.
B) Diazepam has a more rapid onset than lorazepam.
C) Lorazepam has a longer duration of action than does diazepam.
D) Diazepam has a longer duration of action than does lorazepam.
Ans:c

____________________________________________________
77-You are called by the emergency department physician
and asked about your patient from the ambulatory
clinic. The patient is unconscious with continuous
seizures. He has been seizing for 12 minutes.
Which is the most appropriate initial therapy for
this patient?
A. Lorazepam.
41 By :Amera Elfeky Team april 2015
Neurology revision-2015 april

B. Phenobarbital.
C. Propofol.
D. Levetiracetam.
Ans: Answer: A
Benzodiazepines are considered first line because they
can rapidly control seizures. The three most commonly
used benzodiazepines are diazepam, lorazepam, and
midazolam. Lorazepam enters the brain rapidly and
does not partition out quickly, as opposed to diazepam,
which is more lipophilic but partitions out of the brain
rapidly. Answer B, phenobarbital, is an option only after
a benzodiazepine has been administered. Although
its efficacy is similar to that of benzodiazepines, it is
not truly considered first line because of its adverse
reactions. Phenobarbital causes prolonged sedation because
of its long half-life. Answer C, propofol, is not the
best option because it is considered a third-line therapy,
given its adverse reactions. Propofol is associated with
metabolic acidosis syndrome, rhabdomyolysis, renal
failure, and cardiac dysfunction. Answer D, levetiracetam,
is not the best option because it is not a first-line
agent for status epilepticus.
____________________________________________________________
78-Ethosuximide is usually given in divided doses instead of once daily.
Why?
42 By :Amera Elfeky Team april 2015
Neurology revision-2015 april

A. It causes nausea
B. It has a short half-life
C. It has poor bioavailability
D. It is a CYP 3A4 inducer
Ans:a
______________________________________________________
79-T.H. is a 70-year-old man with a long-standing history of generalized
tonic-clonic seizures. He presents to the clinic
today for a follow-up visit after his routine serum laboratory levels were
obtained 4 weeks ago. His liver enzyme test
values were about 10 times the upper limit of normal. The physician
ordered a computed tomography scan (CT) and
liver biopsy. T.H. was given a diagnosis of severe liver disease and
referred back to neurology to review his AEDs.
He is currently taking phenobarbital and valproate. Given his new
diagnosis of liver disease, which is the best recommendation
for treatment?

A. Continue phenobarbital and valproate; no change is needed.


B. Continue phenobarbital and replace valproate with levetiracetam.
C. Replace phenobarbital with levetiracetam and continue valproate.
D. Replace both phenobarbital and valproate with levetiracetam and
pregabalin.
Ans: Answer: D

43 By :Amera Elfeky Team april 2015


Neurology revision-2015 april

T.H. has just been given a diagnosis of severe liver disease.


It is important to attempt to discontinue his drugs
that are metabolized by the liver, both phenobarbital
and valproate. It would be appropriate to begin levetiracetam
and pregabalin, one at a time, because they are
not dependent on the liver for metabolism. Levetiracetam
and pregabalin are both FDA indicated for partialonset
seizures with secondary generalization. When
initiating the drugs for T.H., if levetiracetam controls
his seizures, pregabalin should not be added. AnswerA, Answer B, and
Answer C all contain drug combinations
with AEDs listed that are metabolized by the
liver, making them not the best options for this patient
because of increased laboratory monitoring.
------------------------------------------------------------------------------------------
80-J.D., a 68-year-old man, was seen in your clinic today; upon leaving, he
began to have multiple seizures lasting more
than 10 minutes. During this episode, he did not regain consciousness at
any point. He was taken to the emergency
department, and the physician there wished to begin intravenous
phenytoin. She calls you, asking about drug information
regarding intravenous phenytoin. Which would be best to communicate
to the physician regarding the most
appropriated dosing recommendation for this patient?
A. Phenytoin can be given intramuscularly.

44 By :Amera Elfeky Team april 2015


Neurology revision-2015 april

B. The infusion rate of intravenous phenytoin cannot exceed 50


mg/minute.
C. Intravenous phenytoin should be diluted before reaching the patient.
D. Intravenous phenytoin can cause tissue necrosis upon extravasation.
Answer: B
J.D. is seen in the emergency department for status epilepticus.
The emergency department physician begins
phenytoin intravenously. The maximal infusion rate for
phenytoin is 50 mg/minute, and it is important to communicate
to minimize complications such as hypotension,
cardiac arrhythmias, and venous cording. Answer A,
phenytoin, can be given intramuscularly but only when
intravenous access cannot be achieved because of the
higher occurrence of tissue necrosis with this route of administration.
Answer C, phenytoin, is safest when diluted
in normal saline as opposed to dextrose, which would increase
the likelihood of microcrystal formation and tissue
necrosis. Answer D, tissue necrosis, can occur with intravenous
extravasation and intramuscular injection.
_________________________________________________________

81-K.L. is a 65-year-old man with a new diagnosis of complex partial


seizures. An EEG was performed that shows epileptiform
abnormalities, confirming a diagnosis of epilepsy. K.L. also has a history of
diabetes mellitus and associated peripheral

45 By :Amera Elfeky Team april 2015


Neurology revision-2015 april

neuropathy. His renal function is stable. K.L.’s physician would like a


recommendation for an AED.

1. Given his comorbidities, which drug would be most appropriate for


K.L.?
A. Felbamate.
B. Gabapentin.
C. Lamotrigine.
D. Phenytoin.

2. Through the progression of K.L.’s diabetes, his renal function becomes


severely compromised, and his physician
would like to avoid renally eliminated AEDs. Given this information, which
drug would be best to treat his epilepsy?
A. Topiramate.
B. Lamotrigine.
C. Pregabalin.
D. Vigabatrin.
1.Answer: B
K.L. is older than 60 years and has just been given a
diagnosis of new-onset epilepsy. K.L. also has diabetic
peripheral neuropathy. Gabapentin is a good drug to
start with because it has an FDA-labeled indication for
both complex partial seizures and diabetic peripheral
neuropathy. Veterans Administration (VA) Cooperative

46 By :Amera Elfeky Team april 2015


Neurology revision-2015 april

Study 428 supports the use of gabapentin in patients


who are older than 60 with new-onset seizures with respect
to efficacy and tolerability. Answer A, Answer C,
and Answer D all have indications for partial-onset seizures;
however, they may not be as effective for diabetic
peripheral neuropathy. Felbamate is not considered a
first-line choice for partial-onset epilepsy because of its
adverse event profile
2.Answer: B
K.L. now has severe renal dysfunction, so it would be
appropriate to choose an AED that is metabolized by
the liver and not totally eliminated by the kidneys. Lamotrigine
is effective for partial-onset seizures and has
proved to be a well-tolerated drug by the VA Cooperative
Study 428. Answer A, topiramate is metabolized
through the liver. However, due to severe renal dysfunction
and topiramate's carbonic anhydrase inhibition, the
risk of kidney stones outweigh the potential benefit of
the AED. Answer C and Answer D are AEDs that are
either minimally metabolized or altogether eliminated
by the kidneys. You could choose one of these AEDs;
however, extensive monitoring of renal function and
clinical toxicity would be necessary, making them less
desirable options. Vigabatrin is FDA indicated for infantile

47 By :Amera Elfeky Team april 2015


Neurology revision-2015 april

spasms and refractory complex partial seizures,


making it not the best AED for this patient at this time.
__________________________________________________________

82-Your patient will be admitted for a cholecystectomy.


She is taking carbamazepine 400 mg orally three
times/day. She will be unable to take anything by
mouth for 3 days after surgery and requires an AED
available as an injectable formulation. Which is
the best AED treatment to recommend during this
time (assuming all will provide adequate seizure
control)?
A. Carbamazepine.
B. Levetiracetam.
C. Topiramate.
D. Lamotrigine
Answer: B
Levetiracetam is correct because it is available as an injectable
formulation. Levetiracetam is also available in
oral, tablet, and extended-release formulations. Levetiracetam
is 100% bioavailable, so a one-to-one conversion
between intravenous and oral can be made when
the patient is able to take oral medications. You would
also need to make sure that levetiracetam is effective

48 By :Amera Elfeky Team april 2015


Neurology revision-2015 april

for the patient’s seizure type. Carbamazepine, topiramate,


and lamotrigine are unavailable in an injection
formulation; therefore, they are not the best choices at
this time. Lamotrigine is available as an orally disintegrating
tablet but would be inappropriate for this patient
because the tablets are still being absorbed by the gastrointestinal
system. Levetiracetam is FDA indicated as
adjunctive therapy, so as soon as the patient is taking
oral medications, carbamazepine can be reinitiated, if
necessary.
_____________________________________________________________
83-A 43-year-old business woman has a history of withdrawal seizures
when she stops drinking alcohol. After successful alcohol withdrawal
treatment, she is scheduled to leave the inpatient setting and start her
outpatient substance abuse treatment program. During the inpatient
admission, laboratory values were all within normal limits. Today, she
states that she still really craves alcohol and inquires about potential
treatment options to help her maintain her alcohol abstinence. She plans
to return to work in 2 weeks and wants to avoid taking medications by
mouth.
Which is the best choice to discuss with this patient?
A Naltrexone orally and multivitamin supplementation orally
B Disulfiram orally and folate supplementation orally
C Lorazepam im and B12 im injection once a month
D Naltrexone extended-release injectable solution
Answer D. Evidence supports the use of naltrexone extended-release
injectable to prevent craving for alcohol, and it is administered once a

49 By :Amera Elfeky Team april 2015


Neurology revision-2015 april

month. Thus, this treatment option would help the patient with her goal
to maintain abstinence and avoid taking a medication by mouth. Although
naltrexone and disulfiram, in the oral formulation, are useful in helping
some patients maintain sobriety, it will not help this patient meet her
stated goal. Multivitamin supplementation may include folate, and it
would not be necessary to suggest that folate be included in the
medication regimen. Once a patient has completed detoxification off
alchol, benzodiazepine use should be avoided because patients can
develop dependence upon this category of medications. Unless there is
evidence of a B12 deficiency, there is no obvious reason for this
medication to be included for this medication regimen.
______________________________________________________
84-A 75-year-old nursing home resident with newly diagnosed Parkinson’s
disease and a history of memory problems and confusion is best treated
with:

A) Amantadine
B) Benztropine
C) Pramipexole
D) Carbidopa/levodopa
Ans:d

________________________________________________________
85-G.T. is a 58-year-old male who has very poorly controlled complex
partial seizures. He is currently being evaluated in the emergency
department and is unable to take anything by mouth. Which antiepileptic
drug is not suitable to use in this patient?

A. Phenytoin

50 By :Amera Elfeky Team april 2015


Neurology revision-2015 april

B. Carbamazepine
C. Valproic acid
D. Levetiracetam
F. Both B and D
Ans:b
___________________________________________________
86-Which AED(s) is not associated with weight gain?
A) Zonisamide
B) Vigabatrin
C) Valproic acid
D) Topiramate
E) Both a and d
Ans:e
___________________________________________________
87-A 63-year-old with Parkinson’s disease has done well on rasagiline 1
mg once a day and ropinirole 4 mg 3 times a day for several years. In the
past, higher doses of ropinirole resulted in excessive drowsiness. He now
needs more symptom relief. The best recommendation would be to:
A) Consider surgery
B) Add entacapone
C) Add carbidopa/levodopa
D) Switch ropinirole to pramipexole
Ans:c
_______________________________________________________

51 By :Amera Elfeky Team april 2015


Neurology revision-2015 april

88- Her headache pain is easily relieved with sumatriptan 100 mg orally as
the occasion requires. However, with her last dose, she experienced
substernal chest pain radiating to her left arm. She reported to her local
emergency department, where she had a complete workup. Her final
diagnoses were coronary artery disease and hypertension. For these
conditions, she was placed on hydrochlorothiazide 25 mg orally every
morning. R.M.’s family physician asks which drug is best for R.M. to use
for her migraine headaches.
A. Frovatriptan.
B. Zolmitriptan.
C. Dihydroergotamine.
D. Naproxen.
======
89 If R.M. (from question 16) requires a drug for migraine prophylaxis,
which is best to recommend?
A. Propranolol.
B. Valproic acid.
C. Amitriptyline.
D. Gabapentin
88- Answer: D
The patient with migraines and cardiac disease presents a difficulty in
choice of drug for acute treatment. All triptans and ergotamines are
contraindicated in this situation. A nonsteroidal anti-inflammatory drug is
a possible choice.

89. Answer: A
When possible, a drug for migraine prophylaxis should be selected to
confer additional benefit on a patient for a concomitant disease state. In

52 By :Amera Elfeky Team april 2015


Neurology revision-2015 april

the patient with coronary artery disease and hypertension, propranolol


would be an excellent choice for migraine prevention.
________________________________________________________
90-A 22-year-old woman with a 7 year history of bipolar disorder presents
in the clinic today with complaints of insomnia, low energy, crying
episodes, guilt feelings and suicidal thoughts. She takes no medications at
this time.
Which of the following is the most appropriate to recommend for
treatment of this patient matched with its most common adverse effect?
A Lithium, liver dysfunction
B Fluoxetine/olanzapine, weight reduction
C -Quetiapine, orthostatic hypotension
D Valproic acid, stimulation
Ans:c
___________________________________________________
91. M.G. is a 15-year-old male adolescent with a diagnosis of juvenile
myoclonic epilepsy. He has been prescribed sodium divalproate. On which
adverse effect is it best to counsel M.G.?
A. Oligohidrosis.
B. Renal stones.
C. Alopecia.
D. Word-finding difficulties.
Ans:c-Valproic acid and its derivatives are associated with alopecia. The
hair will grow back if the drug is discontinued and sometimes even if the
drug is continued. There are reports of the regrown hair being curly when
patients previously had straight hair
________________________________________________________

53 By :Amera Elfeky Team april 2015


Neurology revision-2015 april

92. J.G. is a 34-year-old patient who has been maintained on


carbamazepine extended release 400 mg orally two times/day for the
past 2 years. She has had no seizures for the past 4 years. She presents to
the emergency department in status epilepticus. Which drug is best to use
first?
A. Diazepam.
B. Lorazepam.
C. Phenytoin.
D. Phenobarbital
92. Answer: B
Lorazepam is the drug of choice for status epilepticus. It is less lipophilic
than diazepam; therefore, it does not redistribute from the central
nervous system as quickly. After the seizures are stopped with lorazepam,
a long-acting drug (phenytoin, fosphenytoin, or phenobarbital) should be
administered to prevent further seizures.
_________________________________________________________
93. Ethosuximide is usually given in divided doses instead of once daily.
Why?

A. It causes nausea
B. It has a short half-life
C. It has poor bioavailability
D. It is a CYP 3A4 inducer
Ans:. answer A.
Dose-related nausea occurs in up to 40% of patients taking ethosuximide.
Once-daily dosing is still effective, but gastrointestinal effects appear to
be dose-related. Ethosuximide has a relatively long half-life compared to

54 By :Amera Elfeky Team april 2015


Neurology revision-2015 april

other AEDs (60 hrs in adults, 30 hrs in children) and good oral
bioavailability. It is not known to induce or inhibit any hepatic enzymes.
_________________________________________________________
94. S.S. is a 22-year-old woman who has always had episodes of “zoning
out.” Recently, one of these episodes occurred after an examination while
she was driving home. She had a non-injury accident, but it prompted a
visit to a neurologist. She is given a diagnosis of absence seizures. Which
drug is best to treat this type of epilepsy?
A. Phenytoin.
B. Tiagabine.
C. Carbamazepine.
D. Ethosuximide.
. Answer: D
Ethosuximide is useful for absence seizures. The other listed medications
are not used for absence seizures.
_________________________________________________________
95. L.M. is a 43-year-old man who received a diagnosis of progressive-
relapsing multiple sclerosis 2 years ago. He has been taking glatiramer
acetate since then. However, no discernible difference exists in the
number of exacerbations he has experienced. He has spasticity in his legs,
which has caused several falls during the past month, and he experiences
fatigue that worsens as the day progresses. Which drug therapy is best for
L.M.’s multiple sclerosis?
A. Cyclophosphamide.
B. Methylprednisolone.
C. Azathioprine.
D. Mitoxantrone.
=====

55 By :Amera Elfeky Team april 2015


Neurology revision-2015 april

96. Which is best to treat L.M.’s (from question 18) spasticity?


A. Diazepam.
B. Baclofen.
C. Carisoprodol.
D. Metaxalone.
=====
97. Which is best to treat L.M.’s (from question 18) fatigue?
A. Propranolol.
B. Lamotrigine.
C. Amantadine.
D. Ropinirole
ANS: 95. Answer: D
Mitoxantrone is the only one of the given choices with an FDA indication
for the treatment of multiple sclerosis. In addition, it has the best clinical
trial evidence of efficacy. Methylprednisolone is used for treating acute
multiple sclerosis exacerbations. Cyclophosphamide and azathioprine
have been studied in progressive forms of multiple sclerosis, but their
data are not as robust as are those for mitoxantrone.

96. Answer: B
Treatment of spasticity in multiple sclerosis requires the use of a centrally
acting agent. Of the choices given, only diazepam and baclofen are
centrally acting. Because of the significant fatigue and drowsiness seen
with diazepam, baclofen is usually a first-line therapy. Another acceptable
choice would be tizanidine.

97. Answer: C

56 By :Amera Elfeky Team april 2015


Neurology revision-2015 april

Agents used to treat multiple sclerosis–related fatigue include


amantadine and methylphenidate. The other choices are not used in
multiple sclerosis.
___________________________________________________________
98. G.Z., a 26-year-old woman, presents with a 6-month history of
“spells.” The spells are all the same, and all of them start with a feeling in
the abdomen that is difficult for her to describe. This feeling rises toward
the head. The patient believes that she will then lose awareness. After a
neurologic workup, she is given a diagnosis of focal seizures evolving to a
bilateral, convulsive seizure. The neurologist is considering initiating
either carbamazepine or oxcarbazepine. Which is the most accurate
comparison of carbamazepine and oxcarbazepine?
A. Oxcarbazepine causes more liver enzyme induction than
carbamazepine.
B. Oxcarbazepine does not cause rash.
C. Oxcarbazepine does not cause hyponatremia.
D. Oxcarbazepine does not form an epoxide intermediate in its
metabolism.
======
99. When you see G.Z. 6 months later for follow-up, she tells you she is
about 6 weeks pregnant. She has had no seizures since beginning drug
therapy. Which is the best strategy for G.Z.?
A. Immediately discontinue her seizure medication.
B. Immediately discontinue her seizure medication and give folic acid.
C. Continue her seizure medication.
D. Change her seizure medication to phenobarbital.
ANS: 98- Answer: D

57 By :Amera Elfeky Team april 2015


Neurology revision-2015 april

Carbamazepine forms an active epoxide intermediate (carbamazepine-


10,11-epoxide), whereas oxcarbazepine does not. Carbamazepine induces
more liver enzymes than oxcarbazepine. However, hyponatremia is more
closely associated with oxcarbazepine than carbamazepine. Both drugs
can cause allergic rashes.
99. Answer: C
Alterations to seizure treatment regimens can be made when patients
present to the health system before pregnancy. In this case, a different
drug may be chosen, or medications may be eliminated if the patient is
taking more than one seizure medication. In addition, efforts should be
made to maintain the patient on the lowest possible doses that control
seizures. However, when the patient presents to the health system
already pregnant, the current medications are usually continued rather
than risk an increase in seizures during a medication change. Again, the
lowest possible doses that control seizures should be used.
________________________________________________________
100. T.M. is an 18-year-old new patient in the pharmacy where you work.
He presents a prescription for carbamazepine 100 mg 1 orally two
times/day with instructions to increase to 200 mg 1 orally three
times/day. Currently, he does not take any medications and does not
have any drug allergies. During your counseling session, T.M. tells you he
must have blood drawn for a test in 3 weeks. Which common potential
adverse effect of carbamazepine is best assessed through a blood draw?
A. Leukopenia.
B. Renal failure.
C. Congestive heart failure.
D. Hypercalcemia.
======
101. One month later, T.M. returns to your pharmacy with a new
prescription for lamotrigine 25 mg with instructions to take 1 tablet daily

58 By :Amera Elfeky Team april 2015


Neurology revision-2015 april

for 2 weeks, followed by 1 tablet two times/day for 2 weeks, followed by


2 tablets two times/day for 2 weeks, and then 3 tablets two times/day
thereafter. He tells you that he is discontinuing carbamazepine because
he developed a rash a few days ago. Which is the best response?
A. The rash is likely caused by carbamazepine because carbamazepine
rash often has delayed development.
B. The rash is unlikely caused by carbamazepine because carbamazepine
rash usually presents after the first dose.
C. The rash is probably not caused by carbamazepine; it is probably
attributable to carbamazepine-induced liver failure.
D. The rash is unlikely caused by carbamazepine; it is probably
attributable to carbamazepine-induced renal failure.
======
102. T.M. wants to know why it is necessary to increase the dose of
‫اﻟﻔﺟر‬ lamotrigine so slowly. Which reply is best?
A. It causes dose-related psychomotor slowing.
B. It causes dose-related renal stones.
C. It causes dose-related paresthesias.
D. It causes dose-related rash.
100. Answer: A
Leukopenia is a common adverse effect of carbamazepine. Up to 10% of
patients will experience a transient decrease in white blood cell counts;
however, the potential for serious hematologic abnormalities, including
agranulocytosis and aplastic anemia, exists. Complete blood cell counts
are recommended before initiation and periodically during therapy.
101. Answer: A
In general, dermatologic reactions to anticonvulsants occur after a delay
of 2–8 weeks, rather than immediately after medication initiation.

59 By :Amera Elfeky Team april 2015


Neurology revision-2015 april

102. Answer: D
One of the most unusual characteristics of lamotrigine is that the rash
seen with this drug is often related to the speed of titration. Valproic acid
inhibits the metabolism of lamotrigine; therefore, when these drugs are
used together, the lamotrigine titration must be slowed even further.
Psychomotor slowing, renal stones, and paresthesias are associated with
topiramate and zonisamide.
_____________________________________________

103. S.R. is a 37-year-old patient who began taking phenytoin 100 mg 3


capsules orally at bedtime 6 months ago. He has experienced several
seizures since that time; the most recent seizure occurred this past week.
At that time, his phenytoin serum concentration was 8 mcg/mL. The
treating physician increased his dose to phenytoin 100 mg 3 capsules
orally two times/day. Today, which best represents his expected serum
concentration?
A. 10 mcg/mL.
B. 14 mcg/mL.
C. 16 mcg/mL.
D. 20 mcg/mL.
25. Answer: D
Phenytoin shows nonlinear pharmacokinetics. A small increase in dose
may result in a large increase in serum concentration. Therefore, without
performing any calculations, we can surmise that an increase from 300
mg/day to 600 mg/day would more than double the serum concentration.
______________________________________________
104-Typically, carbamazepine is initiated at a low dose.
Which provides the best reason for starting at a low

60 By :Amera Elfeky Team april 2015


Neurology revision-2015 april

dose?
A. Precipitation of absence seizures.
B. Dizziness caused by initial dose.
C. High risk of gastrointestinal adverse effects at
initiation.
D. Autoinduction of the cytochrome P450 (CYP)
system.

Answer D, autoinduction of the CYP, is correct. Carbamazepine


is an autoinducer; therefore, the serum
half-life is longer when the dose is started initially.
Autoinduction takes about 2 weeks for the effect to be
seen, so the half-life will decrease. Answer A is not the
best reason to start at a low dose because carbamazepine
will precipitate absence seizures at most likely any
dose. Answer B, dizziness, is a reason to start at a low
dose, but not the most important reason. Answer C is
one reason to start at a low dose; however, the initial
dose generally does not affect gastrointestinal integrity.
_______________________________________________________

105. What would the phenytoin loading dose be for an 85 kg man?

(A) 300 mg

61 By :Amera Elfeky Team april 2015


Neurology revision-2015 april

(B) 750 mg

(C) 1700 mg

(D) 2000 mg
ANS: 6. Answer: C.
The correct answer is 1700 mg. The loading dose for phenytoin in status
epilepticus is 15–20 mg/kg

106-A 65-year-old patient has had Parkinson’s disease for 10 years and is
currently taking benztropine 2 mg every day, pramipexole 1.5 mg 3 times
a day, and carbidopa/levodopa 25/100 mg 4 times a day. His wife claims
that he is complaining of seeing spiders and bugs running across the floor.
The first thing to do is:
A) Add clozapine
B) Add quetiapine
C) Taper and stop the benztropine
D) Taper and stop the carbidopa/levodopa
ANS:C
______________________________________________
107-A 64-year-old man presents to the emergency department 90
minutes after acute-onset altered mental status, slurred speech, and
right-sided weakness. His medical history is significant for hypertension,
type 2 diabetes mellitus, and atrial fibrillation. The patient’s family states
that he was taking lisinopril, metformin, and warfarin as an outpatient.
Physical examination reveals a blood pressure of 170/95 mm Hg and heart
rate of 89 beats/minute. Ongoing aphasia and right-sided weakness are
confirmed, and his NIH (National Institutes of Health) stroke scale score is
24. An emergency non-contrast head computed tomography is performed

62 By :Amera Elfeky Team april 2015


Neurology revision-2015 april

within 1 hour of presentation, and the results are consistent with an


acute ischemic stroke. Results of urgent laboratory tests show an
international normalized ratio (INR) of 1.4.
Which is the most appropriate therapy for this patient at this time?
A Aspirin 325 mg orally daily
B Intravenous heparin infusion
C Intravenous recombinant tissue plasminogen activator (rTPA)
D Intravenous nicardipine infusion
ANS:C
__________________________________________________________
108-Which of the following is a serious adverse effect of mitoxantrone?
A) Hepatotoxicity
B) Renal failure
C) Skin site reactions
D) Leukemia
ANS:D
________________________________________________________
109-
T or F

If the patient has an additional stroke while taking aspirin,


increasing the aspirin dose to provide additional benefit.
FALSE

________________________________________________

63 By :Amera Elfeky Team april 2015


Neurology revision-2015 april

110-which antiepileptic??
__________ decreases lamotrigine metabolism
ANS:VALPROIC ACID

_________________________________________________
111-This AED is an enzyme inducer, and is associated with hyperactivity,
cognitive impairment and osteoporosis.
ANS:PHENOPARPITAL
_________________________________________________
112-what antiepileptic may induce new onset seizure and status
epilepticus in pts without seizure hx
tiagablin
-------------------------------------------------------------------
113-
T or F
All antiepileptic medications may contribute to osteopenia or
osteoporosis.
True
the medications most often associated with poor bone health are
carbamazepine, clonazepam, phenobarbital, and valproic acid
____________________________________________________
114- A 58-year-old woman with multiple sclerosis presents
to her primary care physician for follow-up. She
ambulates
well and has few issues with her disease.

64 By :Amera Elfeky Team april 2015


Neurology revision-2015 april

Which of the following agents may be beneficial in


slowing the progression of this disease?
(A) Dalfampridine
(B) Fingolimod
(C) Mitoxantrone
(D) Vitamin A
(E) Vitamin E
The answer is B: Fingolimod. Fingolimod is the first
oral drug that can slow the progression of disability
and reduce the frequency and severity of symptoms in
MS, offering patients an alternative to the currently
available injectable therapies. Fingolimod alters
lymphocyte
migration, resulting in sequestration of
lymphocytes in lymph nodes. (A) Dalfampridine, a
potassium channel blocker administered orally, improves
walking speeds versus placebo. (C) Mitoxantrone
is a cytotoxic anthracycline analog, which can kill
T cells. (D) Vitamin A therapy has not been shown to
slow progression of MS. (E) Vitamin E therapy has not
been shown to slow progression of MS.
______________________________________________________

115- A 25-year-old woman has been taking an antiepileptic

65 By :Amera Elfeky Team april 2015


Neurology revision-2015 april

drug for a seizure disorder for 1 year. She now complains


of episodes of blurred vision and diplopia.
Which of the following antiepileptic drugs has she
most likely been taking?
(A) Carbamazepine
(B) Ethosuximide
(C) Lorazepam
(D) Phenobarbital
(E) Valproic acid
The answer is A: Carbamazepine. There are many
drugs used to control or limit seizure activity.
Carbamazepine is a drug that may cause blurred vision
and diplopia with use. Of the drugs listed, carbamazepine
is the most likely culprit behind this
patient’s symptoms. Carbamazepine induces its own
metabolism, so dosage adjustments early in therapy
may be necessary. It is also used to treat trigeminal
neuralgia. (B) Ethosuximide is one of the safer antiepileptic
drugs. It is often used to treat absence seizures.
It does not usually cause visual disturbances.
(C) Lorazepam is a benzodiazepine that blocks seizure
activity by lowering the seizure threshold. Its
most common side effect is sedation and does not
usually cause visual disturbances. (D) Phenobarbital

66 By :Amera Elfeky Team april 2015


Neurology revision-2015 april

is a barbiturate that blocks seizure activity by lowering


the seizure threshold. Its most common side effect is
sedation and does not usually cause visual disturbances.
(E) Valproic acid blocks sodium channels to
prevent the spread of seizure activity. Its most common
side effect is gastrointestinal upset and does not
usually cause visual disturbances.
--------------------------------------------------------------------------------------------

116-A 72-year-old woman with Parkinson’s disease is taking


a medication that increases release of dopamine,
blockade of cholinergic receptors, and inhibiting the
N-methyl-d-aspartate receptor. This describes which
of the following agents?
(A) Amantadine
(B) Bromocriptine
(C) Pramipexole
(D) Rotigotine
(E) Tolcapone
The answer is A: Amantadine. Amantadine has several
effects on several neurotransmitters implicated in
causing parkinsonism, including increasing the release
of dopamine, blockading cholinergic receptors,
and inhibiting the N-methyl-d-aspartate (NMDA)

67 By :Amera Elfeky Team april 2015


Neurology revision-2015 april

type of glutamate receptors. Current evidence supports


an action at NMDA receptors as the primary
action at therapeutic concentrations. (B) Bromocriptine
is a dopamine receptor agonist used in advanced
Parkinson’s disease. (C) Pramipexole is a dopamine
receptor agonist used in advanced Parkinson’s disease.
(D) Rotigotine is a dopamine receptor agonist
used in the treatment of early Parkinson’s disease.
(E) Tolcapone is a nitrocatechol derivative that selectively
and reversibly inhibit COMT.

_______________________________________________________
117 -A 15-year-old boy presents to clinic for follow-up for
his tonic-clonic seizures. He reports that he has not
had a seizure in the past 6 months. However, he has
been more tired recently and is unsure why. A complete
blood count is performed and shows megaloblastic
anemia. The physician told the patient that
this was most likely a side effect of his antiseizure
medication.
What is the most likely medication he
was taking?
(A) Carbamazepine
(B) Ethosuximide

68 By :Amera Elfeky Team april 2015


Neurology revision-2015 april

(C) Phenobarbital
(D) Phenytoin
(E) Valproic acid
The answer is D: Phenytoin. Phenytoin is used in the
treatment of tonic-clonic seizures. Phenytoin has
many side effects, like hirsutism, nystagmus, gingival
hyperplasia, and megaloblastic anemia. (A)
Common side effects of carbamazepine include
ataxia and agranulocytosis, not megaloblastic anemia.
(B) Common side effects of ethosuximide include
GI distress and Stevens–Johnson’s syndrome,
not megaloblastic anemia. (C) Common side effects
of phenobarbital include sedation and ataxia, not
megaloblastic anemia. (E) Common side effects of
valproic acid include
GI distress and hepatotoxicity,
not megaloblastic anemia.

________________________________________________________
118. T.L. is a 35-year-old man with complex partial seizures. He is
otherwise healthy. He was placed on phenytoin after a seizure about 2
months ago. He currently takes phenytoin 100 mg 3 capsules orally every
night. During his clinic visit, he tells you he has had no seizures, and he
has no signs of toxicity. He is allergic to sulfa drugs. His phenytoin serum
concentration is 17.7 mcg/mL. Which is the best interpretation of this
concentration?

69 By :Amera Elfeky Team april 2015


Neurology revision-2015 april

A. It is too low.
B. It is too high.
C. It is just right.
D. A serum albumin concentration is required to interpret this
concentration
118. Answer: C
The therapeutic range for phenytoin is 10–20 mcg/mL. Although a serum
concentration should never be interpreted without clinical information,
this patient is not having any seizures, nor is he experiencing toxicity.
Because he is otherwise healthy, does not have known kidney
dysfunction, and is not elderly, there is no need for an albumin
concentration
_________________________________________________
119. S.S. is a 22-year-old woman who has always had episodes of ―zoning
out.‖ Recently, one of these episodes
occurred after an examination while she was driving home. She had a
non-injury accident, but it prompted
a visit to a neurologist. She is given a diagnosis of absence seizures. Which
drug is best to treat this type of
epilepsy?
A. Phenytoin.
B. Tiagabine.
C. Carbamazepine.
D. Ethosuximide.
Answer: D
Ethosuximide is useful for absence seizures. The other
listed medications are not used for absence seizures.

70 By :Amera Elfeky Team april 2015


Neurology revision-2015 april

----
_____________________________________________________________
_____
120. J.B. is a 25-year-old man with a history of seizure disorder. He has
been treated with phenytoin 200 mg
orally twice daily for 6 months, and his current phenytoin concentration is
6.3 mcg/mL. His neurologist
decides to increase his phenytoin dose to 300 mg twice daily. Which
adverse effect is J.B. most likely to
experience related to the dose increase?
A. Drowsiness.
B. Acne.
C. Gingival hyperplasia.
D. Rash.
Answer: A
Drowsiness is a dose-related adverse effect of phenytoin.
Acne, gingival hyperplasia, and rash can also be adverse
effects, but they are not dose related.
_________________________________________________________
121- M.G., a 15-year-old male adolescent with a diagnosis of juvenile
myoclonic epilepsy, has been prescribed
sodium divalproate. On which adverse effect is it best to counsel M.G.?
A. Oligohidrosis.
B. Renal stones.
C. Alopecia.
D. Word-finding difficulties
Answer: C
Valproic acid and its derivatives are associated

71 By :Amera Elfeky Team april 2015


Neurology revision-2015 april

with alopecia. The hair will grow back if the drug


is discontinued and sometimes even if the drug is
continued. There are reports of the regrown hair being
curly when patients previously had straight hair.
_____________________________________________________________
_____________
Questions 9 and 10 pertain to the following case
122-G.Z., a 26-year-old woman, presents with a 6-month history of
―spells.‖ The spells are all the same, and all start
with a feeling in the abdomen that is difficult for her to describe. This
feeling rises toward the head. The patient
believes that she will then lose awareness. After a neurologic workup, she
is given a diagnosis of focal seizures
evolving to a bilateral, convulsive seizure. The neurologist is considering
initiating either carbamazepine or
oxcarbazepine.
122. Which is the most accurate comparison of carbamazepine and
oxcarbazepine?
A. Oxcarbazepine causes more liver enzyme induction than
carbamazepine.
B. Oxcarbazepine does not cause rash.
C. Oxcarbazepine does not cause hyponatremia.
D. Oxcarbazepine does not form an epoxide intermediate in its
metabolism
Answer: D
Carbamazepine forms an active epoxide intermediate
(carbamazepine-10,11-epoxide), whereas oxcarbazepine
does not. Carbamazepine induces more liver enzymes
than oxcarbazepine. However, hyponatremia is
more closely associated with oxcarbazepine than

72 By :Amera Elfeky Team april 2015


Neurology revision-2015 april

carbamazepine. Both drugs can cause allergic rashes.


_____________________________________________________________
_____________________
Patient Cases (continued)
123. When you see G.Z. 6 months later for a follow-up, she tells you she is
about 6 weeks pregnant. She has had
no seizures since beginning drug therapy. Which strategy is best for G.Z.?
A. Discontinue her seizure medication immediately.
B. Discontinue her seizure medication immediately and give folic acid.
C. Continue her seizure medication.
D. Change her seizure medication to phenobarbital.
Answer: C
Alterations to seizure treatment regimens can be made
when patients present to the health system before
pregnancy. In this case, a different drug may be chosen,
or medications may be eliminated if the patient is taking
more than one seizure medication. In addition, efforts
should be made to maintain the patient on the lowest
possible doses that control seizures. However, when the
patient presents to the health system already pregnant,
the current medications are usually continued, rather
than risk an increase in seizures during a medication
change. Again, the lowest possible doses that control
seizures should be used.
_____________________________________________________________
____
Questions 124–125 pertain to the following case.
L.R. is a 78-year-old man who presents to the emergency department for
symptoms of right-sided paralysis. He
states that these symptoms began about 5 hours ago and have not
improved since then. He also has hypertension,
73 By :Amera Elfeky Team april 2015
Neurology revision-2015 april

benign prostatic hypertrophy, diabetes mellitus, erectile dysfunction, and


osteoarthritis.
124. Which assessment is most accurate regarding L.R.’s risk factors for
stroke?
A. Erectile dysfunction, age, osteoarthritis.
B. Sex, diabetes mellitus, osteoarthritis.
C. Benign prostatic hypertrophy, diabetes mellitus, age, sex.
D. Age, diabetes mellitus, sex, hypertension.
Answer: D
Non-modifiable risk factors for stroke include age, race,
and male sex. Somewhat modifiable risk factors include
hypercholesterolemia and diabetes mellitus. Modifiable
stroke risk factors include hypertension, smoking, and
atrial fibrillation. Less well-documented risk factors
include obesity, physical inactivity, alcohol abuse,
hyperhomocysteinemia, hypercoagulability, hormone
replacement therapy, and oral contraceptives. Modification
of risk factors, if possible, may translate into reduced stroke
risk, which should be a focus of all stroke prevention plans.
_____________________________________________________________
_
125. Is L.R. a candidate for tissue plasminogen activator for treatment of
stroke? Which option is the best response?
A. Yes.
B. No, he is too old.
C. No, his stroke symptoms began too long ago.
D. No, his diabetes mellitus is a contraindication for tissue plasminogen
activator.
Answer: C
Contraindications to administering tissue plasminogen
activator for stroke include intracranial or subarachnoid
74 By :Amera Elfeky Team april 2015
Neurology revision-2015 april

bleeding (or history), other active internal bleeding,


recent intercranial surgery, head trauma, BP greater
than 185/110 mm Hg, seizure at stroke onset, intracranial
neoplasm, atrioventricular malformation, aneurysm,
active treatment with warfarin or heparin, and platelet
count less than 100,000. There is no upper limit on
age. Until recently, there was a strict 3-hour limit for
treating strokes. A recent study suggests this limit can
be increased safely to administer tissue plasminogen
activator 4½ hours after symptom onset with additional
criteria.
_____________________________________________________________
_________
126. L.R. was previously taking no drugs at home. Which choice is the best
secondary stroke prevention therapy
for this patient?
A. Sildenafil.
B. Celecoxib.
C. Aspirin.
D. Warfarin.
Answer: C
All patients experiencing a stroke should be placed on
a drug to prevent future events. Appropriate choices
include the following: aspirin, ticlopidine, cilostazol,
clopidogrel, dipyridamole-aspirin, and warfarin. If the
patient has atrial fibrillation, he or she should be treated
with warfarin, dabigatran, or rivaroxaban. If the patient
does not have atrial fibrillation, warfarin offers no
benefit but has considerable risk compared with aspirin.
Otherwise, any of these drugs are reasonable choices.
_____________________________________________________________
_________________

75 By :Amera Elfeky Team april 2015


Neurology revision-2015 april

127. M.R., a 34-year-old pharmacist, has throbbing right-sided headaches.


She experiences nausea, phonophobia, and photophobia with these
headaches, but no aura. She usually has headaches twice a month. She is
hypertensive and morbidly obese. She takes an ethinyl
estradiol/progestin combination oral contraceptive
daily and hydrochlorothiazide 25 mg/day orally. She has a diagnosis of
migraine headaches. Which medication
is best for prophylaxis of her headaches?
A. Propranolol.
B. Valproic acid.
C. Amitriptyline.
D. Lithium.
Answer: A
A β-blocker is a good choice for a patient with the
coexisting condition of hypertension. Valproic acid
and amitriptyline could both increase weight gain in a
morbidly obese patient. Lithium is used for prophylaxis
of cluster headaches.
_____________________________________________________________
________________
128. S.R. is a 54-year-old female homemaker with squeezing, bandlike
headaches that occur three or four times
weekly. She rates the pain of these headaches as 7/10 and finds
acetaminophen, aspirin, ibuprofen, naproxen,
ketoprofen, and piroxicam only partly effective. She wishes to take a
prophylactic drug to prevent these
tension headaches. Which drug is best for prophylaxis of her headaches?
A. Propranolol.
B. Valproic acid.

76 By :Amera Elfeky Team april 2015


Neurology revision-2015 april

C. Amitriptyline.
D. Lithium.
Answer: C
Amitriptyline is as effective as prophylaxis for tension
headaches. β-Blockers and valproic acid are usually used
for migraine headache prophylaxis, and lithium is used
for prophylaxis of cluster headaches.
_____________________________________________________________
__
129. One of the neurologists you work with read a meta-analysis of
migraine treatments (Oldman AD, Smith
LA, McQuay HJ, et al. Pharmacological treatment for acute migraine:
quantitative systematic review. Pain
2002;91:247-57). He is most interested in the outcome of sustained relief
at 24 hours, but he is confused by
the number-needed-to-treat (NNT) analyses. He shows you the following
table:
NNT Drug
6.6 Ergotamine + caffeine
2.8 Eletriptan 80 mg
5.6 Rizatriptan 10 mg
6.0 Sumatriptan 50 mg
NNT = number needed to treat.
Which statement provides the best interpretation of these data?
A. Eletriptan 80 mg is the most effective agent.
B. Ergotamine plus caffeine is the most effective agent.
C. Eletriptan has the most adverse effects.

77 By :Amera Elfeky Team april 2015


Neurology revision-2015 april

D. Ergotamine plus caffeine has the most adverse effects.


Answer: A
The NNT is a concept used to express the number of
patients it would be necessary to treat to have one patient
experience benefit (or to experience adverse effects, if
looking at harm). It is calculated as NNT = 1/[(% improved
on active therapy) − (% improved on placebo)]. The
NNT is calculated for each treatment and is, therefore,
treatment-specific. Low NNTs indicate high treatment
efficacy. If an NNT of 1 were calculated, it would mean
that every patient on active therapy improved and that no
patient on placebo improved.
_____________________________________________________________
130. B.B. is a 33-year-old woman with a recent diagnosis of multiple
sclerosis. Her neurologist wants you to
discuss with her potential medications to prevent exacerbations. During
the discussion, you find that she
and her husband are planning to have a baby in the next few years and
that she is terrified of needles. Which
choice is best for B.B.?
A. Glatiramer acetate.
B. Mitoxantrone.
C. Teriflunomide.
D. Dimethyl fumarate.
Answer: D
Patients unable to give self-injection because of their
fear of needles should not be given glatiramer acetate,
which is a subcutaneous injection. Mitoxantrone has
significant toxicities, and it is infrequently used at
present to treat multiple sclerosis. In addition, this drug
is pregnancy category X. Teriflunomide may require up
to 2 years for elimination or rapid elimination protocols

78 By :Amera Elfeky Team april 2015


Neurology revision-2015 april

before pregnancy; thus, it would not be a good choice


in this patient. Dimethyl fumarate has no data in
human pregnancy right now and is pregnancy categor y
C. However, this patient should carefully plan her
conception and can discontinue the medication before
pregnancy. Of the available choices, dimethyl fumarate
is the best answer.
_____________________________________________________________
_________
131-
Patients with epilepsy are recommended to begin treatment with
monotherapy using a single appropriate agent. Which patient
situation would meet the criteria to place a patient on polytherapy?
• A. Adherence to the regimen is a concern with monotherapy.
• B. Dose-dependent adverse effects have occurred while on
monotherapy.
• C. Drug-drug interactions will not occur with a second
anticonvulsant agent
• D. Two first-line agents have been tried as monotherapy and failed.
No translation available
answer d
_______________________________________________________
__________
132-
B.V. is a 28-year-old woman brought to your emergencydepartment
for treatment of status epilepticus.She receives lorazepam 4 mg
intravenously with subsequent seizure cessation. Which medicationis
the best next treatment step for B.V.?
A. Topiramate
.B. Phenytoin
.C. Zonisamide
.D. Diazepam.
answer b
_______________________________________________________
_____
133-
W.S. is a 57-year-old man initiated on rasagiline for treatment of his
newly diagnosed Parkinson disease.He develops a cough, body

79 By :Amera Elfeky Team april 2015


Neurology revision-2015 april

aches, and nasal congestion. Which medication is best to treat W.S.’s


symptoms?
A. Guaifenesin
.B. Dextromethorphan
.C. Tramadol.
D. Pseudoephedrine.
answer a
_______________________________________________________
____
134-
A 25-year-old man with a history of seizure disorder. He has been
treated with phenytoin 200 mg orally twice daily for 6 months, and his
current phenytoin concentration is 5.9 mcg/mL. His neurologist
decides to increase his phenytoin dose to 300 mg twice daily. Which
adverse effect is this patient most likely to experience with this
dosage increase?
•A. Acne
•B. Drowsiness
• C. Gingival hyperplasia
• D. Rash
answer b
_______________________________________________________
_
135-
S.R. is a 37-year-old patient who began taking phenytoin 100 mg 3
capsules orally at bedtime 6 months ago. He has experienced several
seizures since that time; the most recent seizure occurred this past
week. At that time, his phenytoin serum concentration was 8 mcg/mL.
The treating physician increased his dose to phenytoin 100 mg 3
capsules orally two times/day. Today, which best represents his
expected serum concentration?
A. 10 mcg/mL
.B. 14 mcg/mL.
C. 16 mcg/mL
.D. 20 mcg/mL.
answer d
______________________________________________________
136-
Patients should be told to drink plenty of fluidwhen taking which of the
following?
80 By :Amera Elfeky Team april 2015
Neurology revision-2015 april

A. Carbamazepine
B. Topiramate
C. Levetiracetam
D. Gabapentin
E. Phenytoin
answer b
__________________________________________________-
137-
J.G. is a 34-year-old patient who has been maintained on
carbamazepine extended release 400 mg orally two times/day for the
past 2 years. She has had no seizures for the past 4 years. She
presents to the emergency department in status epilepticus. Which
drug is best to use first?
A. Diazepam.
B. Lorazepam
.C. Phenytoin
.D. Phenobarbital.
answer b
lorazepam is the drug of choice in emergency status epilepticus
,rapid onset 2-3 minutes ( very very very fast onset )
_______________________________________________________
__________________
138-
B.V. is a 28-year-old woman brought to your emergencydepartment
for treatment of status epilepticus.She receives lorazepam 4 mg
intravenously with subsequent seizure cessation. Which medicationis
the best next treatment step for B.V.?
A. Topiramate.
B. Phenytoin.
C. Zonisamide
.D. Diazepam.
answer b
_______________________________________________________
_______
139-
J.H. is a 42-year-old man with complex partial seizuresfor which he
was prescribed topiramate. Hehas been increasing the dose of
topiramate every other day according to instructions from his
primarycare provider. He comes to the pharmacy where you work but
seems a little confused and has difficulty finding the words to have a
81 By :Amera Elfeky Team april 2015
Neurology revision-2015 april

conversation with you. Which is the best assessment of J.H.’s


condition?
A. Discontinue topiramate; he is having an allergic reaction
.B. Increase his topiramate dose; he is having partial seizures
.C. Slow the rate of topiramate titration; he is having psychomotor
slowing
.D. Get a topiramate serum concentration; he is likely
supratherapeutic.
answer c

___________________________________________________________
140- Which of the following patients may be a candidate for carotid
endarterectomy?
A) An 80-year-old man with a history of TIA and 70% stenosis of the
symptomatic internal carotid artery (ICA)
B) A 46-year-old woman with coronary artery disease and peripheral
vascular disease, no history of TIA or stroke, with 45% stenosis of the L
ICA and occlusion of the R ICA.
C) A patient with atrial fibrillation and an intolerance to warfarin.
D) A 65-year-old man with a history of weakness on the R that lasted less
than 10 minutes. He has 40% stenosis of his L ICA and 60% of his R ICA.
E) None of the above.
Ans:a
AED IN PREGNANCY:
all the 'old' antiepileptic drugs - barbiturates, phenytoin, carbamazepine,
but very little is known of the risk of the 'new' antiepileptic drugs such as
levetiracetam, topiramate, oxcarbazepine, gabapentin, pregabalin,
tiagabine and zonisamide.
AEDs such as valproate and phenobarbital were associated with a higher
risk of major malformations than newer AEDs such as lamotrigine and

82 By :Amera Elfeky Team april 2015


Neurology revision-2015 april

levetiracetam. Topiramate was associated with an increased risk of cleft


lip compared with that of a reference population.
SO .lamotrigine are safe to use and better to use it with folic acid
But in a limited dose
Levitracitam , zonisamide, oxycarbazine ..safe to use but with monitoring
Ethosuccinamide categoty c in pregnancy ..it is mainly used in pediatrics
..but using it in pregnancy showed worsening of some cases
It is better to avoid it in pregnancy
Valproic acid avoided in first trimester
Monotherapy is more prefer able than combination

________________________________________________________
141- Thrombolytic therapy of acute ischemic stroke is indicated:
A) Prior to arrival in the ER
B) Only with streptokinase
C) If initiated within at least 12 hours of the acute event
D) If CT scan is negative for intracranial hemorrhage
E) Even though AHA guidelines are not closely followed
Ans:d
_____________________________________________________________
______
142-S.R is 37 years old pt who began taking phenytoin 100mg 3 cap at bed
time 6 months ago..he experienced several seizures ,..at that time his
serum conc was 8mcg/ml.the physician increase the dose to 100mg 3 cap
twice daily,..What is the expected serum conc??
1-10mcg/ml
2-14 mcg/ml
3-16 mcg/ml
83 By :Amera Elfeky Team april 2015
Neurology revision-2015 april

4-20 mcg/ml
Ans:4
______________________________________________
143-
Which of the following is tre regarding phenytoin?
A. The maximum rate of intravenous administration is 50 mg/min
B. If intravenous access can't be established,phenytoin can be given
1M
C. Because phenytoin contains propylene glycolit is soluble is any iv
fluid
D. It is an inhibitor of the cytochrome P450systemE. A major
limitation to the use of the productin pediatric patients is the lack of
acommercially available liquid formulation
answer a
_______________________________________________________
____
144-
Which of the following medications may causeseizures in an adult
patient with renal failure?
A. Meperidine
B. Phenobarbital
C. Carbamazepine

84 By :Amera Elfeky Team april 2015


Neurology revision-2015 april

D. Lamotrigine
E. Theophylline
answer a
_______________________________________________________
_______
145-
T.M. is an 18-year-old new patient in the pharmacy where you work.
He presents a prescription for carbamazepine100 mg 1 orally two
times/day with instructions to increase to 200 mg 1 orally three
times/day. Currently, he does not take any medications and does not
have any drug allergies. During your counseling session, T.M. tells
you he must have blood drawn for a test in 3 weeks. Which common
potential adverse effect of carbamazepine is best assessed through a
blood draw?
A. Leukopenia
.B. Renal failure
.C. Congestive heart failure
D. Hypercalcemia.
answer a
_______________________________________________________
_______
146-
A 20 year old lady presents with ahistory of a single tonic-clonic
seizure. She hadnot been taking any illicit drugs. She alsoreports the
occurrence of occasional absentspells and brief jerking of his upper
limbs,when she has been out late partying.Which of the Following
drugs is most appropriate?
1-Lorazepam
2-Carbamazepine
3-Phenytoin
4-Gabapentin
5-Sodium valproate
answer 5
_______________________________________________________
______
147-
A patient on which of the following medications should be made
aware of the importans of good oral hygiene
a-felbamate
b-phenytoin
85 By :Amera Elfeky Team april 2015
Neurology revision-2015 april

c-zonisamide
d-phenobarbital
e-levetiracetam
answer b

_______________________________________________________
___________
148- J.H is 42 years old man with a complex partial seizures & he is taking
topiramate ,..he has been increasing the dose every other day,acc to
instruction from his primary care provider ,..he came th the pharmacy
alittle confused with difficulty to find the words to have a conversation
,..Which is the best assessment ...
1-discontinue topiramate ,..he is having an allergic reaction..
2-increase the topiramate dose ,.he is having partial seizure ...
3-slow the rate of titration,..he is having psychomotor slowing..
4-Get a topramate serum conc ,..he is supratherapeutic...
Ans:3
___________________________________________________
148-Stroke prevention with warfarin therapy is cost-effective in:
A) Patients with a history of TIA
B) Carotid occlusion of 60% in one vessel
C) Patients with atrial fibrillation and >1 high-risk factor
D) Those where intracerebral bleeding rates are less than 0.1%.
E) Primary prevention in non-AF patients
Ans:c
_________________________________________________
149-Which of the following is the most common cause of acute neurologic
deterioration of an ischemic stroke patient in the first 3 days after the
event?

86 By :Amera Elfeky Team april 2015


Neurology revision-2015 april

A) Pulmonary embolism
B) Pneumonia
C) Cerebral edema
D) Recurrent ischemia
E) Dementia
Ans:c
________________________________________________________
150-All the following are primary features of Parkinson’s disease except:
A) Convulsions
B) Rigidity
C) Rest tremor
D) Slowness of movement
Ans:a
______________________________________________________
151-A 42-year-old African American male is admitted to the hospital 8
hours after experiencing the onset of right-sided weakness and difficulty
with speech. He has a past medical history of hypertension for 10 years
and obesity.
Medson admission: none
Review of systems: 6’1”, 280 lb, BP = 190/100, P = 90, EKG = NSR
Neurological exam: mild R leg and arm weakness, expressive aphasia
Which of the following acute therapies has been shown to improve long-
term outcome in a patient like the one presented above?
A) Aspirin
B) Subcutaneous heparin

87 By :Amera Elfeky Team april 2015


Neurology revision-2015 april

C) Enoxaparin
D) Warfarin
Ans:a
_______________________________________________________
152-. B.V. is a 28-year-old woman brought to your emergency
department for treatment of status epilepticus. She receives lorazepam 4
mg intravenously with subsequent seizure cessation. Which medication
is the best next treatment step for B.V.?
A. Topiramate.
B. Phenytoin.
C. Zonisamide.
D. Diazepam.

. Answer: B
In general, medications to treat status epilepticus should be in parenteral
formulation to facilitate rapid administration. Once the seizures of status
epilepticus have been stopped, a second, long-acting drug should be
started to prevent seizure recurrence. Medications typically used for this
purpose include phenytoin, fosphenytoin, phenobarbital, and
(sometimes) valproic acid. There is no need to administer another
benzodiazepine because the seizure activity has ceased.
________________________________________________________
153-which of the following drugs cause prerenal AKI
1-valsartan
2lithium
3-gentamycin
4-cyclosporin
Ans:1
_________________________________________________

88 By :Amera Elfeky Team april 2015


Neurology revision-2015 april

154-ph=7.45 and pco2=55 and hco3=45 what is this state


1-compensated metabolic alkalosis
2-compensated respiratory alkalosis
3-non-compensated metabolic alkalosis
4-non-compensated respiratory alkalosis
Ans:1
_______________________________________________
155-Which characteristic(s) of the patient described in Question #4
has/have been shown to independently increase her risk of developing a
symptomatic intracerebral hemorrhage after t-PA?
A) Systolic blood pressure < 170 mmHg?
B) Negative CT
C) Age <80
D) Severe stroke (NIHSS <20)
E) b and d
Answer d
___________________________________________________
156-Which of the following antiplatelet medications is associated with a
high incidence of headache?
A) Warfarin
B) Aspirin
C) Clopidogrel
D) Aspirin + dipyridamole
E) Unfractionated heparin
Ans:d

89 By :Amera Elfeky Team april 2015


Neurology revision-2015 april

________________________________________________________
157-A patient with Parkinson’s disease is taking carbidopa/levodopa
25/100 mg three times a day, and reports that he tends to slow down an
hour or two before his next carbidopa/levodopa dose. This patient is
most likely experiencing:
A) Delayed onset response
B) Dyskinesia
C) Freezing
D) Wearing off
Ans:d
__________________________________________________
158-entacapone is useful because it:
A) Is a MAO-B inhibitor
B) Is a D2 receptor agonist
C) Blocks peripheral COMT
D) Inhibits dopa decarboxylase
Ans:c
______________________________________________________
159-All the following can produce parkinsonian symptoms except:
A) Apomorphine
B) Haloperidol
C) Metoclopramide
D) Prochlorperazine
Ans:a
______________________________________________________

90 By :Amera Elfeky Team april 2015


Neurology revision-2015 april

160-An 85-year-old patient taking carbidopa/levodopa 25/100 mg 3 times


a day for Parkinson’s disease is experiencing end-of-dose wearing off. The
best next step is to:
A) Add tolcapone
B) Consider surgery
C) Add trihexyphenidyl
D) Increase carbidopa/levodopa to 4 times daily
Ans;d
______________________________________________________
161. VS is a 24-year-old woman with secondarily generalized partial
seizures and moderate mental retardation. She is currently experiencing
8–10 seizures/month and is taking phenytoin 200 mg PO TID. She is 88
pounds above her ideal body weight. Because of her obesity, which one of
the following medications should be avoided?
(A) carbamazepine
(B) valproic acid
(C) topiramate
(D) zonisamide
. Answer: B.
Weight gain is an adverse effect of valproic acid.
_____________________________________________________________
162. J.H. is a 42-year-old man with complex partial seizures for which
he was prescribed topiramate. He has been increasing the dose of
topiramate every other day according to instructions from his primary
care provider. He comes to the pharmacy where you work but seems a
little confused and has difficulty finding the words to have a
conversation with you. Which is the best assessment of J.H.’s condition?
A. Discontinue topiramate; he is having an allergic reaction.
B. Increase his topiramate dose; he is having partial seizures.
C. Slow the rate of topiramate titration; he is having psychomotor
slowing.
D. Get a topiramate serum concentration; he is likely supratherapeutic.
91 By :Amera Elfeky Team april 2015
Neurology revision-2015 april

. Answer: C
Psychomotor slowing is a very troublesome adverse effect for many
patients initiated on topiramate. It usually manifests as difficulty
concentrating, difficulty thinking, word-finding difficulties, and a feeling
of slowness of movement. The usual dosage titration for topiramate calls
for increasing the dose every week. This patient has been increasing the
dose of topiramate every other day. Because psychomotor slowing is
related to the speed of titration, this makes slowing the rate of titration
the most probable answer. Partial seizures could present as confusion;
however, they are unlikely to be a continuous condition.
__________________________________________________________
163. T.M. is an 18-year-old new patient in the pharmacy where you work.
He presents a prescription for carbamazepine 100 mg 1 orally two
times/day with instructions to increase to 200 mg 1 orally three
times/day. Currently, he does not take any medications and does not
have any drug allergies. During your counseling session, T.M. tells you he
must have blood drawn for a test in 3 weeks. Which common potential
adverse effect of carbamazepine is best assessed through a blood draw?
A. Leukopenia.
B. Renal failure.
C. Congestive heart failure.
D. Hypercalcemia.

. Answer: A
Leukopenia is a common adverse effect of carbamazepine. Up to 10% of
patients will experience a transient decrease in white blood cell counts;
however, the potential for serious hematologic abnormalities, including
agranulocytosis and aplastic anemia, exists. Complete blood cell counts
are recommended before initiation and periodically during therapy.

92 By :Amera Elfeky Team april 2015


Neurology revision-2015 april

____________________________________________________________
164. One month later, T.M. returns to your pharmacy with a new
prescription for lamotrigine 25 mg with instructions to take 1 tablet daily
for 2 weeks, followed by 1 tablet two times/day for 2 weeks, followed by
2 tablets two times/day for 2 weeks, and then 3 tablets two times/day
thereafter. He tells you that he is discontinuing carbamazepine because
he developed a rash a few days ago. Which is the best response?
A. The rash is likely caused by carbamazepine because carbamazepine
rash often has delayed development.
B. The rash is unlikely caused by carbamazepine because carbamazepine
rash usually presents after the first dose.
C. The rash is probably not caused by carbamazepine; it is probably
attributable to carbamazepine-induced liver failure.
D. The rash is unlikely caused by carbamazepine; it is probably
attributable to carbamazepine-induced renal failure.

. Answer: A
In general, dermatologic reactions to anticonvulsants occur after a delay
of 2–8 weeks, rather than immediately after medication initiation.
________________________________________________________
165. J.B. is a 25-year-old man with a history of seizure disorder. He has
been treated with phenytoin 200 mg orally two times/day for 6 months,
and his current phenytoin concentration is 6.3 mcg/mL. His neurologist
decides to increase his phenytoin dose to 300 mg two times/day. Which
adverse effect is J.B. most likely to experience related to the dose
increase?
A. Drowsiness.
B. Acne.
C. Gingival hyperplasia.

93 By :Amera Elfeky Team april 2015


Neurology revision-2015 april

D. Rash.

. Answer: A
Drowsiness is a dose-related adverse effect of phenytoin. Acne, gingival
hyperplasia, and rash can also be adverse effects, but they are not dose
related.
_____________________________________________________________
166. True. 58% of patients who develop a rash with phenytoin will also
get a rash with carbamazepine. 40% of patients who get a rash with
carbamazepine will also get one with phenytoin, while 20-30% will get
one with oxcarbazepine. 80% of patients who experience a rash with
phenobarbital will get a rash with carbamazepine or phenytoin. Patients
with a history of developing a rash with other AEDs are more likely to get
one with lamotrigine as well
. Which dopamine agonist is contraindicated with ondansetron?
A. Mirapex (pramipexole)
B. Requip (ropinirole)
C. Apokyn (apomorphine)
D. Azilect (rasagiline)

answer. C. 5HT3 antagonists such as ondansetron may enhance the


hypotensive effects of apomorphine and is contraindicated. If used,
patients should be closely monitored for increased hypotension.
Ondansetron may be used safely with patients currently taking either
pramipexole or ropinirole. Rasagiline is an MAO-B inhibitor, not a
dopamine agonist.
_____________________________________________
167- Match the following for treatment of MS.
1. Can experience flu-like symptoms that usually dissipate after 2-3
months

94 By :Amera Elfeky Team april 2015


Neurology revision-2015 april

2. Risk of progressive multifocal leukoencephalopathy


3. Cardiotoxicity is lifetime dose limiting toxicity
4. Used for acute relapse of MS

a. Prednisone
b. Beta Interferons
c. Natalizumab
d. Mitoxantrone

44. 1. B 2. C 3. D 4. A
_____________________________________________________
168. Which of the following medications has an elevated risk of
hepatoxicity in children?
a. oxcarbazepine
b. valproate
c. topiramate
d. ethosuximide

. b. valproate
________________________________________________________
169. A 16-year-old female adolescent has been well controlled on
valproic acid for a history of myoclonic seizures. During her yearly
physical examination, she complains of menstrual irregularities and
excessive weight gain. Which is the best alternative to valproic acid
for the treatment of this patient’s myoclonic seizures?
A. Oxcarbazepine
B. Levetiracetam
C. Topiramate
D. Lamotrigine

answer. C. Topiramate

95 By :Amera Elfeky Team april 2015


Neurology revision-2015 april

Topiramate is the most reasonable choice because it has the added


benefit of weight loss.
170. A 25-year-old man with a history of seizure disorder. He has
been treated with phenytoin 200 mg orally twice daily for 6 months,
and his current phenytoin concentration is 5.9 mcg/mL. His
neurologist decides to increase his phenytoin dose to 300 mg twice
daily. Which adverse effect is this patient most likely to experience
with this dosage increase?
A. Acne
B. Drowsiness
C. Gingival hyperplasia
D. Rash
B. Drowsiness
Acute, idiosyncratic, and chronic adverse effects are a concern with
antiepileptic drugs. The older antiepileptic drugs tend to have similar
neurologic adverse effects that include sedation, drowsiness,
dizziness, and ataxia. Phenytoin can have central nervous system
(CNS)-depressant effects including lethargy, fatigue, blurred vision,
incoordination, and drowsiness. Clinicians would need to monitor for
these effects in patients initiating the drug or with dosage changes.
171. L.R. is a 78-year-old man who presents to the emergency
department for symptoms of right-sided paralysis. He states that these
symptoms began about 5 hours ago and have not improved since then.
He also has hy-pertension, benign prostatic hypertrophy, diabetes
mellitus, erectile dysfunction, and osteoarthritis. Which is the most
accurate assessment of L.R.’s risk factors for stroke?
A. Erectile dysfunction, age, osteoarthritis.
B. Sex, diabetes mellitus, osteoarthritis.
C. Benign prostatic hypertrophy, diabetes mellitus, age, sex.
D. Age, diabetes mellitus, sex, hypertension.

. Answer: D
Non-modifiable risk factors for stroke include age, race, and male sex.
Somewhat modifiable risk factors include hypercholesterolemia and
diabetes mellitus. Modifiable stroke risk factors include hypertension,
smoking, and atrial fibrillation. Less well-documented risk factors include
obesity, physical inactivity, alcohol abuse, hyperhomocysteinemia,

96 By :Amera Elfeky Team april 2015


Neurology revision-2015 april

hypercoagulability, hormone replacement therapy, and oral


contraceptives. Modification of risk factors, if possible, may translate into
reduced stroke risk, which should be a focus of all stroke prevention
plans.
_______________________________________________________
172. L.S. is taking carbidopa/levodopa 25 mg/100 mg orally four
times/day and trihexyphenidyl 2 mg orally three times/day for his
Parkinson disease. L.S.’s wife reports that he is often confused and
experiences con-stipation; he has trouble talking because of his dry
mouth. Which change is best to resolve these symptoms?
A. Increase carbidopa/levodopa.
B. Increase trihexyphenidyl.
C. Decrease carbidopa/levodopa.
D. Decrease trihexyphenidyl.

. Answer: D
Anticholinergic drugs (benztropine and trihexyphenidyl) commonly cause
adverse effects such as confusion, dry mouth, urinary retention, and
constipation in older patients. Decreasing or eliminating these drugs may
resolve the difficulties.
_________________________________________________
173. C.A., a 57-year-old white man who just retired from the New York
City Fire Department, has been experi-encing tremors in his right hand
that have become progressively worse for the past 6 months. He has dif-
ficulty walking. He also has backaches and no longer plays golf. In
addition, he is losing his sense of taste. He is given a diagnosis of
Parkinson disease. Which is the best treatment for this man?
A. Trihexyphenidyl.
B. Entacapone.
C. Apomorphine.

97 By :Amera Elfeky Team april 2015


Neurology revision-2015 april

D. Ropinirole.

. Answer: D
Ropinirole, a direct dopamine agonist, is a good choice for initial
treatment in a patient with Parkinson disease. Trihexyphenidyl would
control his tremor but would not help his difficulty walking, which likely
represents bradykinesia. Entacapone is a catechol-O-methyl-transferase
inhibitor; it should only be used in conjunction with carbidopa/levodopa.
Apomorphine is for severe on-off symptoms.

174- M.R., a 34-year-old pharmacist, has throbbing right-sided


headaches. She experiences nausea, phonopho-bia, and
photophobia with these headaches, but no aura. She usually has
headaches two times/month. She is hypertensive and morbidly
obese. She takes an ethinyl estradiol/progestin combination oral
contraceptive daily and hydrochlorothiazide 25 mg/day orally. She
has a diagnosis of migraine headaches. Which medica-tion is
best for prophylaxis of her headaches?
A. Propranolol.
B. Valproic acid.
C. Amitriptyline.
D. Lithium.
. Answer: A A β-blocker is a good choice for a patient with the coexisting
condition of hypertension. Valproic acid and amitriptyline could both
increase weight gain in a morbidly obese patient. Lithium is used for
prophylaxis of cluster headaches.
175. S.R. is a 54-year-old female homemaker with squeezing,
bandlike headaches that occur three or four times/ week. She
rates the pain of these headaches as 7 of 10 and finds
acetaminophen, aspirin, ibuprofen, naprox-en, ketoprofen, and
piroxicam only partly effective. She wishes to take a prophylactic
drug to prevent these tension headaches. Which is best for
prophylaxis of her headaches?
A. Propranolol.
B. Valproic acid.
C. Amitriptyline.
D. Lithium
98 By :Amera Elfeky Team april 2015
Neurology revision-2015 april

. Answer: C Amitriptyline is effective as prophylaxis for tension headaches.


β-Blockers and valproic acid are usually used for migraine headache
prophylaxis, and lithium is used for prophylaxis of cluster headaches.
176.S.F. is a 33-year-old African American woman of Cuban descent living
in the Miami area. This morning, her right leg became progressively
weaker over about 3 hours. She was previously healthy except for a bro-
ken radius when she was 13 years old and a case of optic neuritis when
she was 25 years old. Which is the best method for treating S.F.’s
exacerbation?
A. Interferon beta-1a.
B. Glatiramer acetate.
C. Mitoxantrone.
D. Methylprednisolone.

. Answer: D
Methylprednisolone is the only option used for treating acute
exacerbations. Other options would be high-dose oral prednisone or
adrenocorticotropic hormone. Interferon beta-1a, glatiramer acetate, and
mitoxantrone are all used as disease-modifying therapies.
_______________________________________________________

99 By :Amera Elfeky Team april 2015

You might also like